SCI Q & A Review Flashcards

1
Q
1.  Regarding epidemiology, which of the following is the most common cause of spinal cord injury (SCI)?
       A)  Falls
       B)  Violence
       C)  Motor vehicle accidents
       D)  Sports/recreation
A

C) Automobile accidents are the leading cause of SCI. However, there are differences among age groups. Falls are reported as the most common cause in the elderly, and violence is the leading cause in African Americans.

How well did you know this?
1
Not at all
2
3
4
5
Perfectly
2
Q
2.  What is the leading cause of death in chronic spinal cord injury (SCI)?
       A)  Heart disease
       B)  Respiratory disease
       C)  Genitourinary disease
       D)  Suicide
A

B) The leading cause of death in spinal cord injury patients is respiratory diseases, with pneumonia as the most common cause.

How well did you know this?
1
Not at all
2
3
4
5
Perfectly
3
Q

According to the International Standards for the Neurologic Classification of Spinal Cord Injury (ISNCSCI), American Spinal Injury Association (ASIA) impairment score of B is defined as?
A) Motor preservation greater than 3 levels below neurologic level and greater than half the key muscles below the single neurologic level are less than 3
B) No motor function more than 3 levels below the motor level with sensory preservation including sacral sparing
C) Motor preservation greater than 3 levels below the neurologic level and greater than half the key muscles below the single neurologic level are graded greater than or equal to 3
D) Complete injury with no sensory or voluntary anal sphincter contraction

A

B) • A = Complete. No sensory or motor preservation in sacral segments, S4–5.
• B = Sensory incomplete. Sensory preservation below the neurological level with sacral sparing. No motor function more than 3 levels below the motor level.
• C = Motor incomplete. Motor preservation below the neurologic level (> 3 levels) and greater than half key muscles below the single neurologic level graded less than 3.
• D = Motor incomplete. Motor preservation below the neurologic level (> 3 levels) and greater than half muscles below the single neurologic level graded greater than or equal to 3.
• E = Sensory and motor exams are normal.

How well did you know this?
1
Not at all
2
3
4
5
Perfectly
4
Q
4.  A spinal cord injured patient was noted to have the following on physical examination: motor preservation greater than 3 levels below the neurologic level and greater than half the key muscles below the single neurologic level were less than 3. This would be classified as an American Spinal Injury Association (ASIA) level:
       A)  ASIA A
       B)  ASIA B
       C)  ASIA C
       D)  ASIA D
A
  1. C) • A = Complete. No sensory or motor preservation in sacral segments, S4–5.
    • B = Sensory incomplete. Sensory preservation below the neurological level with sacral sparing. No motor function more than 3 levels below the motor level.
    • C = Motor incomplete. Motor preservation below the neurologic level (> 3 levels) and greater than half key muscles below the single neurologic level graded less than 3.
    • D = Motor incomplete. Motor preservation below neurologic level (> 3 levels) and greater than half muscles below the single neurologic level graded greater than or equal to 3.
    • E = Sensory and motor exams are normal.
How well did you know this?
1
Not at all
2
3
4
5
Perfectly
5
Q
  1. Which of the following are key muscles tested in the scoring of the American Spinal Injury Association (ASIA) exam?
    A) C5-biceps, C7-triceps, T1-adductor digiti minimi, L5-quadriceps, S1-flexor hallucis longus
    B) C5-biceps, C6-flexor carpi ulnaris, C8-flexor digitorum profundus, L3-quadriceps, L4-adductor magnus
    C) C5-biceps, C8-flexor digitorum superficialis, L2-sartorius, L3-quadriceps, L5-biceps femoris
    D) C5-biceps, C8-flexor digitorum profundus, T1-abductor digiti minimi, L4-tibialis anterior, S1-gastrocnemius
A

D) There are 10 key muscles tested, which include the following:
• C5 = biceps brachialis; elbow flexors
• C6 = extensor carpi radialis; wrist extensors
• C7 = triceps; elbow extensors
• C8 = flexor digitorum profundus; finger flexor of middle finger
• T1 = abductor digiti minimi; small finger abductor
• L2 = iliopsoas; hip flexors
• L3 = quadriceps; knee extensors
• L4 = tibialis anterior; ankle dorsiflexors
• L5 = extensor hallucis longus; long toe extensors
• S1 = gastrocnemius; ankle plantar flexors

How well did you know this?
1
Not at all
2
3
4
5
Perfectly
6
Q
  1. Which of the following is true regarding the zone of partial preservation (ZPP)?
    A) Classified only in incomplete injuries
    B) The most rostral segment below the assigned level that has sensory or motor function
    C) The motor ZPP does not follow the sensory ZPP
    D) All of the above
A
  1. C) The ZPP is defined as the most caudal dermatomes and myotomes below the sensory and motor levels that remain partially innervated. It is classified only in complete injuries. The motor ZPP does not follow the sensory ZPP. In the scoring sheet, enter the motor or sensory level if there are no segments below and N/A for incomplete injuries. To calculate the length, count the number of levels from the sensory or motor level to the ZPP level.
How well did you know this?
1
Not at all
2
3
4
5
Perfectly
7
Q
7.  A hyperextension injury that occurs in low-velocity trauma that affects the upper (greater than the lower) extremities is called?
       A)  Central cord syndrome
       B)  Brown-Séquard syndrome
       C)  Anterior cord syndrome
       D)  Cauda equina syndrome
A

A) Central cord syndrome is an incomplete injury that results in lower motor neuron weakness at the level of injury and upper motor neuron spasticity below the injury. It typically occurs in the elderly who have a preexisting spondylosis. Brown-Séquard is a hemisection of the cord that results in ipsilateral motor and proprioceptive loss with contralateral loss of pain and temperature. Anterior cord syndrome occurs with a vascular injury that results in loss of motor and pain/temperature sensation with preservation of light touch and position sense. Cauda equina occurs with burst fractures or central disc herniations resulting in lower motor neuron flaccid paralysis with loss of sensation in the lower lumbar and sacral segments.

How well did you know this?
1
Not at all
2
3
4
5
Perfectly
8
Q
  1. What is the typical presentation of an individual with Brown-Séquard syndrome?
    A) Ipsilateral motor loss at the level of the lesion and contralateral loss of position sense, pain and temperature below the level of the lesion
    B) Contralateral proprioceptive and motor loss at the level of the lesion with ipsilateral loss of pain and temperature below the level of the lesion
    C) Ipsilateral motor and proprioceptive loss at and below the level of the lesion with contralateral loss of pain and temperature below the lesion
    D) Contralateral motor loss at the level of the lesion, ipsilateral proprioceptive loss at the level of the lesion, and contralateral loss of pain and temperature below the lesion
A

C) The neurological findings seen in Brown-Séquard are based on where various pathways travel and cross over in relation to the brainstem and spinal cord. Since it is a hemisection of the cord, the following is seen:
• Ipsilateral motor and proprioceptive loss at and below the level of lesion
• Ipsilateral sensory loss at the level of the lesion
• Contralateral loss of pain and temperature below the level of the lesion.

How well did you know this?
1
Not at all
2
3
4
5
Perfectly
9
Q
  1. Which of the following is true of the recommendations regarding prevention of upper limb pain and injury in spinal cord injury (SCI) patients?
    A) Minimize frequency of repetitive upper limb tasks
    B) Minimize the force used to perform upper limb tasks
    C) Minimize extreme positions of the joints
    D) All of the above
A
  1. D) Task frequency can be modified by limiting the number of transfers or switching to power mobility in high-risk patients. Minimizing force can be achieved by using a lighter wheelchair, improving propulsion techniques, and maintaining an optimal weight. Individuals are encouraged to avoid extreme positions of the wrist, avoid positioning above the shoulder by implementing assistive devices/technology, and avoid extreme shoulder internal rotation and abduction to prevent mechanical impingement.
How well did you know this?
1
Not at all
2
3
4
5
Perfectly
10
Q
  1. What is the recommended duration for anticoagulant prophylaxis for an uncomplicated complete spinal cord injury?
    A) 6 weeks
    B) 8 weeks
    C) 12 weeks
    D) Until discharge from rehabilitation
A
  1. B) Anticoagulant with either low-molecular-weight heparin or adjusted dose unfractionated heparin should be initiated within 72 hours of injury if there is no contraindication. It should be continued for 8 weeks in an uncomplicated case and 12 weeks or until discharge from rehabilitation for those individuals with other risk factors such as lower limb fracture, history of cancer/thrombosis, age > 70, heart failure, or obesity even in those with inferior vena cava filters.
How well did you know this?
1
Not at all
2
3
4
5
Perfectly
11
Q
  1. Vena cava filter placement should be considered for spinal cord injured patients with which of the following?
    A) High cervical cord injury with poor cardiopulmonary reserve
    B) As a substitute for prophylaxis in a complete injury
    C) Older than 70 years
    D) All of the above
A
  1. A) Vena cava placement is indicated in those who have failed prophylaxis, have a contradiction to anticoagulation, and high lesions with poor cardiopulmonary reserve. It is not a substitute for prophylaxis and may increase the risk of complications in the future such as cava thrombosis or filter migration.
How well did you know this?
1
Not at all
2
3
4
5
Perfectly
12
Q
  1. Which of the following is a true statement regarding women with spinal cord injury (SCI) and pregnancy?
    A) The likelihood of pregnancy is reduced since amenorrhea is common
    B) The risk of complications during pregnancy is equivalent to that in the able bodied population
    C) A caesarian section is the preferred method of delivery
    D) Autonomic dysreflexia may be the only presentation of labor in injuries above the T6 level
A

D) Amenorrhea may occur after injury, but most often menstruation returns 6 months after injury. Reproductive function and fertility is unaffected once menstruation returns. These women should be educated about the issues related to pregnancy in SCI, including complications such as increased risk of urinary tract infections, changes in respiratory function, and biomechanical effects of being in a wheelchair. Since uterine innervations arise from T10 to T12, these individuals may not present with the typical symptoms of labor and must be aware of autonomic dysreflexia. Although caesarian sections may be more common in this population, it is not the preferred plan for delivery.

How well did you know this?
1
Not at all
2
3
4
5
Perfectly
13
Q
  1. In the treatment of erectile dysfunction in spinal cord injured males, the physiatrist should discuss which of the following options?
    A) Phosphodiesterase type 5 inhibitors
    B) Intracavernosal injections
    C) Vacuum devices
    D) All of the above
A
  1. D) The physiatrist should begin the discussion of sexual function early and be respectful of this sensitive subject. PDE-5 inhibitors have been successful and well tolerated. Its use is contraindicated with the concurrent use of nitrates. When oral medications are ineffective, an injection of alprostadil can be given, but priapism can occur. A vacuum device requires manual dexterity and is contradicted in those on blood thinners or with a history of sickle cell. Other options include intraurethral medications (which are not as widely available) and implantable penile prostheses (which are not preferred because they can lead to corporal tissue destruction).
How well did you know this?
1
Not at all
2
3
4
5
Perfectly
14
Q
14.  What level spinal cord injury leads one to be concerned about the risk of developing autonomic dysreflexia?
       A)  T4 and above
       B)  T6 and above
       C)  T8 and above
       D)  T10 and above
A

. B) Autonomic dysreflexia occurs as a result of unopposed sympathetic discharge above the major splanchnic outflow (which occurs at T6 through L2). It occurs after spinal shock, when reflexes return. The patient may present with flushing and diaphoresis above the level of the lesion, hypertension, bradycardia, piloerection, skin pallor, and/ or headache.

How well did you know this?
1
Not at all
2
3
4
5
Perfectly
15
Q
15.  What is the most common cause of autonomic dysreflexia?
       A)  Pressure ulcers
       B)  Fractures or other trauma
       C)  Restrictive clothing
       D)  Bladder distention
A

D) The most common causes of autonomic dysreflexia involve the bladder and bowel. There are a number of causes, all of which involve noxious stimuli below the level of the spinal cord injury. All of the above are potential causes, as well as infection, pregnancy, sexual intercourse, diagnostic medical procedures, deep venous thrombosis, and ingrown toenails.

How well did you know this?
1
Not at all
2
3
4
5
Perfectly
16
Q
  1. A T4 American Spinal Injury Association (ASIA) C individual in your office begins to get flushed. You check the individual’s blood pressure and find that it is elevated with a decreased pulse. What should you do next?
    A) Sit the person up and loosen any clothing
    B) Catheterize the individual
    C) Perform fecal disimpaction
    D) Administer nifedipine
A

A) There are several causes for autonomic dysreflexia, but the first action should be to sit the person up and loosen any restrictive clothing, which will lower blood pressure by pooling blood in the lower extremities. Then, it is important to determine the stimulus causing the autonomic dysreflexia. For instance, the most common cause is bladder distention—check for kinks or flush the catheter. Pharmacologic management may be necessary if symptoms persist.

How well did you know this?
1
Not at all
2
3
4
5
Perfectly
17
Q
  1. Which of the following statements is true regarding prognosis and spinal cord injury (SCI)?
    A) Preserved sacral sensation has a better prognosis for lower extremity recovery
    B) Individuals older than 50 years have a better prognosis for recovery
    C) A sensory incomplete injury has a less than 10% chance for ambulation
    D) Muscles with antigravity strength can recover two grades in the first year
A

A) The recovery is faster and more favorable in incomplete injuries and in younger patients. To achieve antigravity strength, it can take an average of 2 months in a complete injury compared with 2 weeks in an incomplete injury. Muscles with a grade of 1 or 2 have a greater probability of improving one grade by 1 year compared with muscles with no activity. ASIA D has the best prognosis for ambulation. Of those persons with ASIA C, 75% will regain the ability to ambulate and 50% in ASIA B. The most significant predictor is the preservation of sacral sensation.

How well did you know this?
1
Not at all
2
3
4
5
Perfectly
18
Q
18.  Which of the following individuals with spinal cord injury (SCI) can transfer on level surface without the use of a board?
       A)  C4
       B)  C5
       C)  C6
       D)  C7
A

D) Expected functional outcomes are important to discuss with an acutely injured patient and his or her family. For transfers, patients with a lesion at C4/C5 usually require total assist with a board or a mechanical lift, C6 patients require minimal assist to independent with the use of a board or lift, and C7 patients are usually independent with or without the board on level surfaces (and some assist to independence on uneven terrain). An individual should gain complete independence in transfers at C8-T1, which was not listed as a choice.

How well did you know this?
1
Not at all
2
3
4
5
Perfectly
19
Q
19.  Which of the following is a risk factor for developing depression post-spinal cord injury (SCI)?
       A)  Male gender
       B)  Age of onset above 40
       C)  Prior history of depression
       D)  Supportive social structure
A

C) Screening for depression should be performed on the initial visit and subsequent follow-up appointments. Owing to limited research, the number of those suffering with depression after their injury may be underrepresented. The following are general risk factors: prior or family history of depression, age of onset below 40, chronic pain, female gender, poor support system, multiple comorbidities, substance abuse, and other major stresses. An individual with a previous depressive episode has a 50% probability of a second event. In the SCI population, the completeness and associated medical issues such as a brain injury heighten the risk of developing depression.

How well did you know this?
1
Not at all
2
3
4
5
Perfectly
20
Q
20.  In a patient with a spinal cord injury, which of the following is not an effective evacuation technique in a bowel program?
       A)  Push-ups
       B)  Abdominal massage
       C)  Supine position
       D)  Deep breathing exercises
A

C) The use of certain maneuvers may aid in evacuation, including push-ups, abdominal massage, a forward leaning position, breathing exercises, and drinking warm fluids. In addition, the upright position can also stimulate motility more effectively than attempting a bowel program while in bed.

How well did you know this?
1
Not at all
2
3
4
5
Perfectly
21
Q

In a spinal cord injured (SCI) patient, which of the following is not true when establishing a bowel program?
A) Schedule a routine the same time of the day after a meal
B) In areflexic bowel, the goal is firm-formed stool that can be manually evacuated
C) Fiber should be uniformly started in each patient
D) A mini-enema can trigger reflexic bowel by acting as mucosal stimulus

A

C) The establishment of a regular and predictable bowel routine is a crucial part of postinjury management. The type of program may depend on whether the bowels are reflexic or areflexic. The bowel program should be scheduled at the same time daily—usually a half hour after a meal to stimulate the gastrocolic reflexes. For the reflexic bowel, the goal is to use a water-soluble lubricant along with suppository to create soft-formed stool that can be evacuated with digital stimulation. For areflexic bowel, the goal is firm-formed stool that can be manually evacuated. A mini-enema, which is docusate, glycerin, and polyethylene glycol, has been shown to minimize the time from medication insertion to evacuation. The decision to incorporate fiber in the diet should take into consideration the individual bowel pattern and is not recommended to be automatically started in every SCI patient.

How well did you know this?
1
Not at all
2
3
4
5
Perfectly
22
Q
What is the classification of a pressure ulcer with full thickness skin loss involving subcutaneous tissue and extending into but not through fascia?
       A)  Stage I
       B)  Stage II
       C)  Stage III
       D)  Stage IV
A

. C) It is vital to learn the classification of pressure ulcers. Stage I involves changes in skin temperature, tissue consistency, and sensation. The skin appears red or pigmented. Stage II is a partial thickness skin loss involving the epidermis, dermis, or both. Stage III involves full thickness tissue loss, but muscle, bone, and tendon are not affected. Stage IV is similar to Stage III, in that there is full thickness involvement but it extends to muscle, bone, tendon, and joint capsule. If eschar is present, then it cannot be staged and is labeled unstageable.

How well did you know this?
1
Not at all
2
3
4
5
Perfectly
23
Q

In a patient with spinal cord injury (SCI), intermittent catheterization should be considered in which of the following?
A) Small bladder capacity (

A

C) Intermittent catheterization is a treatment option for neurogenic bladder. It should be avoided in individuals who are unable to catheterize because of poor hand strength, a caregiver who is unwilling to assist, urethral abnormalities, small bladder capacity, high fluid intake, and the tendency to develop autonomic dysreflexia. Intermittent catheterization can lead to the development of urinary tract infections, stones, incontinence, urethral trauma, and autonomic dysreflexia. Routine urologic follow-up is crucial

How well did you know this?
1
Not at all
2
3
4
5
Perfectly
24
Q

In a patient with spinal cord injury (SCI), suprapubic catheterization should be considered for which of the following?
A) Sacral pressure ulcer
B) Urethral abnormalities or obstruction
C) Improved body image
D) All of the above

A

D) Suprapubic catheterization is an alternative to those individuals who cannot perform intermittent catheterization because of the following reasons: urethral stricture or obstruction, perineal skin breakdown, prostatitis, urethritis, or epididymo-orchitis. It can be considered to improve body image and sexual function. It is the preferred method in the acute phase in patients with urethral trauma.

How well did you know this?
1
Not at all
2
3
4
5
Perfectly
25
Q
25.  In patients with spinal cord injury (SCI), what are the long-term complications of an indwelling catheter?
       A)  Bladder and kidney stones
       B)  Hydronephrosis
       C)  Pyelonephritis
       D)  All of the above
A

D) Indwelling catheters may be an option for a higher level of injury (such as complete tetraplegia). Although reports have shown that the risk of urinary tract infections is greater with an indwelling catheter compared with intermittent catheterization, there are studies that have shown that the risk may be analogous. It should be considered in individuals with elevated detrusor pressures who are at risk for upper tract complications. It is associated with an increased risk of bladder/kidney stones, epididymitis, urinary tract infections, incontinence, pyelonephritis, hydronephrosis, and cancer. Therefore, more frequent cystoscopic evaluation is warranted in these patients.

How well did you know this?
1
Not at all
2
3
4
5
Perfectly
26
Q
  1. Which statement is true regarding the use of alpha blockers in the treatment of detrusor sphincter dyssynergia?
    A) Urethral resistance is increased with the use of alpha blockers
    B) Phosphodiesterase inhibitors should be used with caution in patients on alpha blockers
    C) Alpha blockers should be taken in the morning in the upright position
    D) All of the above
A

B) Detrusor sphincter dyssynergia is a common bladder condition seen in spinal cord injury patients. The detruor is overactive and spastic, and the internal sphincter is also hyperactive. It results in a small bladder that is unable to empty. This increases the risk of high-voiding pressures and vesicoureteral reflux. Alpha receptors are found in the proximal urethra and bladder neck, and therefore alpha blockers can lower urethral resistance. One of the complications of using this medication is orthostatic hypotension. The medication should be taken at night while in the supine position. Patients taking alpha blockers should be cautioned to avoid phosphodiesterase inhibitors to prevent an abrupt drop in blood pressure.

How well did you know this?
1
Not at all
2
3
4
5
Perfectly
27
Q
The external urethral sphincter is innervated by the:
       A)  Hypogastric nerve
       B)  Pelvic nerve
       C)  Vagus nerve
       D)  Pudendal nerve
A

D) The external urethral sphincter is innervated by the pudendal nerve (S2-S4). The internal urethral sphincter is innervated by the hypogastric nerve (T11-L2) and allows for the storage of urine. The parasympathetic system, through the pelvic splanchnic nerve (S2-S4), promotes bladder contraction and voiding.

How well did you know this?
1
Not at all
2
3
4
5
Perfectly
28
Q
28.  What is the most common location of heterotopic ossification (HO) in spinal cord injury (SCI) patients?
       A)  Hip
       B)  Knee
       C)  Shoulder
       D)  Elbow
A

. A) HO is the formation of true bone in ectopic sites that restricts range of motion. HO can present with swelling, fever, limited mobility, or pain. Ninety percentage of the time, in spinal cord injured patients, it occurs in the hips. Serum alkaline phosphatase will be elevated, but is not a specific measure and levels gradually diminish with maturation. HO may not be visible on plain films in the acute phase, but will be seen on bone scan. Treatment includes gentle ranging exercises, etidronate, and rarely radiation therapy. Surgical resection can be considered in severe cases after maturation.

How well did you know this?
1
Not at all
2
3
4
5
Perfectly
29
Q
29.  Which of the following is a risk factor for the development of heterotopic ossification (HO) in spinal cord injury (SCI)?
       A)  Gender
       B)  Level of lesion
       C)  Spasticity
       D)  Race
A

C) The risk of HO is greater in complete spinal cord injuries, older individuals, in the presence of spasticity, and in patients with pressure ulcers. No relationship has been shown with gender, race, level, or cause of injury.

How well did you know this?
1
Not at all
2
3
4
5
Perfectly
30
Q
  1. Which of the following is true of calcium metabolism in spinal cord injury (SCI)?
    A) Hypercalcemia occurs more commonly in females, incomplete paraplegia, and chronic injury
    B) The risk of fractures is comparable to the able-bodied population
    C) Passive weight-bearing (standing with the use of adaptive equipment) results in improved bone mineral density
    D) All of the above
A

C) In SCI, there is a disparity between bone formation and resorption. This results in a greater risk of fractures from osteoporosis, hypercalcemia, and hypercalciuria. Hypercalcemia is seen most often with recent injury, male gender, complete injury, tetraplegia, dehydration, and immobilization. Chronic SCI patients may develop vitamin D deficiency, which requires supplementation. Restricting calcium intake is not recommended. Treatment includes bisphosphonates, intravenous fluids, or calcitonin. Functional electrical stimulation and weight bearing may decrease bone loss.

How well did you know this?
1
Not at all
2
3
4
5
Perfectly
31
Q
The most caudal end of the spinal cord is at which level?
       A)  The 12th thoracic vertebra
       B)  The 10th thoracic vertebra
       C)  The 4th lumbar vertebra
       D)  The 2nd lumbar vertebra
A

D) Up through the third month of fetal life, the spinal cord occupies the whole length of the vertebral canal. After the third month, the rate of lengthening of the spinal cord is slower than the lengthening of the vertebral column.

How well did you know this?
1
Not at all
2
3
4
5
Perfectly
32
Q
32.  How many cervical nerve roots are there?
       A)  5
       B)  6
       C)  7
       D)  8
A
  1. D)
How well did you know this?
1
Not at all
2
3
4
5
Perfectly
33
Q
33.  The C6 nerve root exits:
       A)  Above the C5 vertebra
       B)  Above the C6 vertebra
       C)  Below the C7 vertebra
       D)  Above the C4 vertebra
A
  1. B) In the cervical region, nerves exit the intervertebral foramina just rostral to the vertebra of the same name with the exception of the C8 nerve root, which has no corresponding vertebral body. It resides below C7 and above T1.
How well did you know this?
1
Not at all
2
3
4
5
Perfectly
34
Q
34.  Which of the following is the most common cause of traumatic spinal cord injury (SCI)?
       A)  Motor vehicle crash
       B)  Sports
     C)  Violence
       D)  Falls
A
  1. A) Motor vehicle crashes account for 47% of traumatic SCIs.
How well did you know this?
1
Not at all
2
3
4
5
Perfectly
35
Q
35.  What is the most common level of spinal cord injury (SCI)?
       A)  T10
       B)  T6
       C)  L5
       D)  C5
A
  1. D)
How well did you know this?
1
Not at all
2
3
4
5
Perfectly
36
Q
  1. The lateral spinothalamic tracts:
    A) Control voluntary muscle activity
    B) Transmit proprioception only
    C) Transmit pain and temperature
    D) Transmit proprioception, fine touch, and vibration
A
  1. C) Spinocerebellar tracts transmit unconscious proprioception from the ipsilateral side of the body.
    Lateral corticospinal tracts control voluntary muscle activity.
    Dorsal columns transmit proprioception, fine touch, and vibration sense from the ipsilateral side of the body.
How well did you know this?
1
Not at all
2
3
4
5
Perfectly
37
Q
  1. Which of the following is the most common cause of death for persons with a spinal cord injury (SCI)?
    A) Heart disease
    B) Disease of the respiratory system
    C) Cancer
    D) Stroke
A
  1. B) Twenty-two percentage of deaths for persons with SCI treated at a model systems site were due to diseases of the respiratory system. The next most common cause is heart disease (11.8%), followed by infectious diseases (10.4%).
How well did you know this?
1
Not at all
2
3
4
5
Perfectly
38
Q
38.  In the American Spinal Injury Association (ASIA) examination, the C5 myotome correlates with what muscle group?
       A)  Elbow extensors
       B)  Finger abductors
       C)  Wrist extensors
       D)  Elbow Flexors
A
38.  D) The Key Muscle Groups in the ASIA examination for the upper extremities are as follows:
C5 = Elbow flexors
C6 = Wrist extensors
C7 = Elbow extensors
C8 = Long finger flexors
T1 = Finger abductors
How well did you know this?
1
Not at all
2
3
4
5
Perfectly
39
Q
39.  In the American Spinal Injury Association (ASIA) examination, the C7 myotome correlates with what muscle group?
       A)  Elbow flexors
       B)  Long finger flexors
       C)  Elbow extensors
       D)  Wrist extensors
A
39.  C) The Key Muscle Groups in the ASIA examination for the upper extremities are as follows:
C5 = Elbow flexors
C6 = Wrist extensors
C7 = Elbow extensors
C8 = Long finger flexors
T1 = Finger abductors
How well did you know this?
1
Not at all
2
3
4
5
Perfectly
40
Q
40.  In the American Spinal Injury Association (ASIA) examination, the nipple line is the key dermatome for what level?
       A)  T4
       B)  T10
       C)  T6
       D)  L4
A
  1. A) The T4 dermatome includes the nipple. The T6 level is the xiphoid, L4 is the medial malleolus, and T10 is the umbilicus.
How well did you know this?
1
Not at all
2
3
4
5
Perfectly
41
Q
41.  In the American Spinal Injury Association (ASIA) examination, the umbilicus is the key dermatome for what level?
       A)  T6
       B)  T4
       C)  T10
       D)  L4
A
  1. C) The T10 dermatome includes the umbilicus. The T6 level is the xiphoid, L4 is the medial malleolus, and T4 is the nipple.
How well did you know this?
1
Not at all
2
3
4
5
Perfectly
42
Q
  1. A complete spinal cord injury is defined as:
    A) A transection of the spinal cord
    B) No motor or sensory function preserved in the sacral segments S4–5
    C) No motor sparing, but sensory sparing below the level of injury
    D) Strength less than antigravity below the level of injury
A

B) (ASIA A classification)

How well did you know this?
1
Not at all
2
3
4
5
Perfectly
43
Q
A patient sustains a spinal cord injury. He has the following motor examination:
C5—5/5 bilateral
C6—5/5 bilateral
C7—3/5 bilateral
C8—2/5 bilateral
T1—2/5 bilateral
L1—1/5 bilateral
L2—1/5 bilateral
L3—1/5 bilateral
L4—0/5 bilateral
L5—0/5 bilateral
Sensation—Intact to pinprick and light touch to the armpit; impaired (1/2) from armpit to rectum with pinprick and light touch sparing at S4–5 and rectal tone.
 What is the motor level of injury for this individual?
       A)  C6
       B)  T1
       C)  L3
       D)  C7
A
  1. D) The motor level of injury is defined as the lowest key muscle that has a grade of at least 3, provided the key muscles above that level are graded as 5.
    The sensory level of injury is the most caudal dermatome to have normal (score of 2) sensation for both pinprick and light touch.
    The ASIA Classification System is as follows:
    A = Motor and sensory complete—no sacral sparing including pin prick (PP) or light touch (LT) at any of the S4–5 dermatomes
    B = Sensory incomplete
    C = Motor incomplete—defined as
    1) Sacral sparing of motor function (anal contraction)
    2) Sacral sparing of sensation with motor function present in more than three levels below the motor level on either side (may include nonkey muscles)
    Less than half of muscles 3/5 or greater below the motor level of injury
    D = Motor incomplete—as ASIA C with more than half 3/5 or greater
    E = Neurologically intact
How well did you know this?
1
Not at all
2
3
4
5
Perfectly
44
Q
A patient sustains a spinal cord injury. He has the following motor examination:
C5—5/5 bilateral
C6—5/5 bilateral
C7—3/5 bilateral
C8—2/5 bilateral
T1—2/5 bilateral
L1—1/5 bilateral
L2—1/5 bilateral
L3—1/5 bilateral
L4—0/5 bilateral
L5—0/5 bilateral
Sensation—Intact to pinprick and light touch to the armpit; impaired (1/2) from armpit to rectum with pinprick and light touch sparing at S4–5 and rectal tone.
What is the sensory level of injury?
       A)  T1
       B)  T4
       C)  S5
       D)  C7
A
  1. A) The motor level of injury is defined as the lowest key muscle that has a grade of at least 3, provided the key muscles above that level are graded as 5.
    The sensory level of injury is the most caudal dermatome to have normal (score of 2) sensation for both pinprick and light touch.
    The ASIA Classification System is as follows:
    A = Motor and sensory complete—no sacral sparing including pin prick (PP) or light touch (LT) at any of the S4–5 dermatomes
    B = Sensory incomplete
    C = Motor incomplete—defined as
    1) Sacral sparing of motor function (anal contraction)
    2) Sacral sparing of sensation with motor function present in more than three levels below the motor level on either side (may include nonkey muscles)
    Less than half of muscles 3/5 or greater below the motor level of injury
    D = Motor incomplete—as ASIA C with more than half 3/5 of greater
    E = Neurologically intact
How well did you know this?
1
Not at all
2
3
4
5
Perfectly
45
Q
A patient sustains a spinal cord injury. He has the following motor examination:
C5—5/5 bilateral
C6—5/5 bilateral
C7—3/5 bilateral
C8—2/5 bilateral
T1—2/5 bilateral
L1—1/5 bilateral
L2—1/5 bilateral
L3—1/5 bilateral
L4—0/5 bilateral
L5—0/5 bilateral
Sensation—Intact to pinprick and light touch to the armpit; impaired (1/2) from armpit to rectum with pinprick and light touch sparing at S4–5 and rectal tone.
What is the American Spinal Injury Association (ASIA) Classification for this patient?
       A)  ASIA A
       B)  ASIA B
       C)  ASIA C
       D)  ASIA D
A
  1. C) The motor level of injury is defined as the lowest key muscle that has a grade of at least 3, provided the key muscles above that level are graded as 5.
    The sensory level of injury is the most caudal dermatome to have normal (score of 2) sensation for both pinprick and light touch.
    The ASIA Classification Systeem is as follows:
    A = Motor and sensory complete—no sacral sparing including pin prick (PP) or light touch (LT) at any of the S4–5 dermatomes
    B = Sensory incomplete
    C = Motor incomplete—defined as
    1) Sacral sparing of motor function (anal contraction)
    2) Sacral sparing of sensation with motor function present in more than three levels below the motor level on either side (may include nonkey muscles)
    Less than half of muscles 3/5 or greater below the motor level of injury
    D = Motor incomplete—as ASIA C with more than half 3/5 of greater
    E = Neurologically intact
How well did you know this?
1
Not at all
2
3
4
5
Perfectly
46
Q
46.  An 82-year-old man trips and falls. On presentation, he has an ecchymosis on his chin. On physical examination, bilateral upper extremities were found to have 2/5 strength with elbow flexion and wrist extension, 3/5 strength with elbow extension, finger flexion, and finger abduction, and 4/5 strength in bilateral lower extremities. He has intact sensation. What is this spinal cord injury (SCI) syndrome?
       A)  Brown-Séquard
       B)  Central cord
       C)  Anterior cord
       D)  Cauda equina
A
  1. B) Central cord syndrome is the most common of the incomplete spinal cord lesions. It produces motor weakness greater in the arms than in the legs and variable sensory loss. There is sacral sensory sparing. This syndrome is due to an injury to the central part of the cervical spinal cord.
How well did you know this?
1
Not at all
2
3
4
5
Perfectly
47
Q
  1. A 17-year-old female is stabbed in the back and presents as follows:
    Loss of sensation, paralysis, and loss of vibration below T5 on the left; loss of pain and temperature below T5 on the right. What syndrome does she have?
    A) Brown-Séquard
    B) Central cord
    C) Anterior cord
    D) Cauda equina
A
  1. A) Brown-Séquard syndrome involves an injury to the transverse section of the spinal cord (relative hemisection). The resultant injury involves ipsilateral motor and proprioception loss with contralateral loss of pain and temperature.
How well did you know this?
1
Not at all
2
3
4
5
Perfectly
48
Q

NM 1. Which of the following motor neuron diseases typically causes both upper and lower motor neuron signs?
A) Spinal muscle atrophy II
B) Primary lateral sclerosis (PLS)
C) Amyotrophic lateral sclerosis (ALS)
D) Poliomyelitis

A

C) Weakness may be due to upper or lower motor neuron loss. Upper motor neuron (UMN) signs include weakness, spasticity, hyperreflexia, and upgoing plantar response. Lower motor neuron (LMN) signs include weakness, atrophy, flaccidity, hyporeflexia, and fasciculations. ALS patients with UMN pathology will often have a loss of dexterity or feeling of stiffness in their limbs. Spasticity may further exacerbate weakness and loss of function. This is due to the involvement of the vestibulospinal and reticulospinal tracts. LMN symptoms in the ALS population include muscle weakness, with some muscle fasciculations, atrophy, and muscle cramping. Cramping of abdominal or other trunk muscles should prompt a clinician to strongly consider ALS as a possible diagnosis. Primary lateral sclerosis (PLS) is classified as an UMN lesion. Spinal muscle atrophy II (SMA II) and poliomyelitis are classified as LMN lesions.

How well did you know this?
1
Not at all
2
3
4
5
Perfectly
49
Q

NM 2. A 60-year-old man without any significant past medical history presents to your outpatient office with asymmetric atrophy, weakness, and fasciculations. He also complains of some difficulty swallowing his meals and complains of a strained and strangled quality in his speech. He describes normal bowel and bladder function. Which of the following is most likely his diagnosis?
A) Amyotrophic lateral sclerosis (ALS)
B) Spinal muscle atrophy III
C) Primary lateral sclerosis (PLS)
D) Poliomyelitis

A

A) ALS most commonly affects people in the age group of 40 to 60 years, and the mean age of onset is around 60 years. Onset is usually insidious and painless. Asymmetric weakness is the most common presentation. Dysphagia (oral, pharyngeal), dysarthria, drooling, and aspiration can occur and are signs and symptoms representing bulbar muscular weakness. Also, strained, strangled quality of speech, reduced rate, and low pitch indicate a spastic dysarthria. Bowel and bladder function is typically spared in ALS

How well did you know this?
1
Not at all
2
3
4
5
Perfectly
50
Q

NM 3. Which of the following is considered a poor prognostic factor in patients with amyotrophic lateral sclerosis (ALS)?
A) Predominance of upper motor neuron (UMN) findings at diagnosis
B) Long period from symptom onset to diagnosis
C) Younger age of onset
D) Pulmonary dysfunction early in the clinical course

A

D) Poor prognostic factors include predominance of lower motor neuron (LMN) findings at diagnosis, short period from symptom onset to diagnosis, and older age at time of onset. Bulbar and pulmonary dysfunction early in the disease course is also a poor prognostic factor. Women typically present with bulbar symptoms, as compared with men. It is important to note that electrodiagnostic indicators of poor prognostic indicators include profuse spontaneous fibrillations, positive sharp waves, and low-amplitude compound muscle action potential.

How well did you know this?
1
Not at all
2
3
4
5
Perfectly
51
Q
NM  4.  What is the prognosis for patients with amyotrophic lateral sclerosis?
       A)  50% die within 3 years
       B)  100% die within 3 years
       C)  50% live up to 7 years
       D)  50% live up to 10 years
A
  1. A) The overall median 50% survival rate is 2.5 years after diagnosis. Survival rate is largely dependent on a patient’s decision to use mechanical ventilation and/or a feeding tube, but the 5-year survival rate is between 4% and 30%. Around 10% will live for 10 years.
How well did you know this?
1
Not at all
2
3
4
5
Perfectly
52
Q
NM 5.  Which of the following pharmacologic agents has been approved for patients with amyotrophic lateral sclerosis (ALS) to slow the progression and improve survival?
       A)  Levodopa
       B)  Riluzole
       C)  Baclofen
       D)  Rebif
A

B) This is an antiglutamate agent that may be effective in slowing the disease, prolonging ventilator time, and may improve survival in patients with bulbar onset disease. However, side effects can include asthenia, and the medication is expensive. Nonpharmacological management of ALS includes rehabilitation, preventing contractures, submaximal exercise, tracheostomy, and respiratory therapy. Rebif is a beta interferon that is used to modify the course in multiple sclerosis patients. Baclofen is a derivative of gamma-aminobutyric acid (GABA) and is primarily used to treat spasticity. Levodopa is a medicine used to control symptoms of Parkinson’s disease. Levodopa does not slow the disease process, but it improves muscle movement and delays severe disability.

How well did you know this?
1
Not at all
2
3
4
5
Perfectly
53
Q

NM 6. Which of the following motor neuron diseases typically causes lower motor neuron signs and has the earliest disease onset?
A) Kugelberg–Welander disease
B) Primary lateral sclerosis (PLS)
C) Amyotrophic lateral sclerosis (ALS)
D) Werdnig–Hoffmann disease

A

D) Upper motor neuron signs include weakness, spasticity, hyperreflexia, and upgoing plantar response. Lower motor neuron signs include atrophy, flaccidity, hyporeflexia, and fasciculations. Werdnig–Hoffmann disease is also known as spinal muscular atrophy type I, or acute infantile-onset SMA. This severe disorder often results in death by the age of 2, and disease onset is 3 to 6 months. Recent studies have shown that there is a reported increase in longevity, most likely a result of better overall medical management. However, SMA type I carries the worst prognosis of all the forms of SMA, as patients may never be able to reach the childhood milestone of sitting independently. The disease course is rapid and fatal secondary to respiratory failure. SMA II and III have later onset, and progression is generally slower. Kugelberg–Welander disease is also called SMA type III. All forms of spinal muscle atrophy are classified as lower motor diseases. PLS is an upper motor neuron disease, and ALS is classified as both upper and lower lesion motor disease.

How well did you know this?
1
Not at all
2
3
4
5
Perfectly
54
Q

NM 7. A 16-year-old male presents to your office with concerns that recently he uses his hands and arms to “walk” up his own body from a squatting position. He states that he was otherwise independent with standing and walking and has been doing well as a student and plans on attending college. Which lower motor neuron disease does he most likely have?
A) Spinal muscular atrophy (SMA) type I
B) SMA type II
C) SMA type III
D) Amyotrophic lateral sclerosis (ALS)

A

C) SMA type III (spinal muscular atrophy) is also known as Kugelberg–Welander disease. The disease onset is later than that of type I and II, occurring between 2 and 15 years of age. Patients typically live a normal life expectancy. The progression is slower than the other two variants. Patients usually achieve independent standing/walking. This patient presents with Gower’s sign, which can be seen in SMA type III patients because proximal weakness is greater than distal weakness. These patients typically have normal intelligence. Complications in SMA III are less frequent, but may include hand tremor, tongue fasciculations (late onset), and areflexia. All forms of spinal muscle atrophy are classified as lower motor diseases, and ALS is classified as both upper and lower lesion motor disease.

How well did you know this?
1
Not at all
2
3
4
5
Perfectly
55
Q
NM  8.  A patient with spinal muscular atrophy (SMA) type II (chronic Werdnig–Hoffmann) can usually achieve which of the childhood milestone below?
       A)  Assisted sitting
       B)  Independent sitting
       C)  Independent standing
       D)  Independent ambulatio
A

B) SMA type II is also known as chronic Werdnig–Hoffmann. The disease onset is between 2 and 12 months, and death (often by respiratory failure) occurs by 10 years of age. These patients can usually achieve milestones including independent sitting. They may be able to stand or walk with an assistive device. Answer choice D is therefore incorrect and refers to someone with SMA type III. SMA type I patients never attain the ability to sit independently, and this severe disorder usually results in death by the age of 2.

How well did you know this?
1
Not at all
2
3
4
5
Perfectly
56
Q

NM 9. Which of the following motor neuron diseases has the best prognosis?
A) Kugelberg–Welander disease
B) Chronic Werdnig–Hoffmann
C) Werdnig–Hoffmann disease
D) Both B and C, as their clinical course is similar

A
  1. A) Spinal muscular atrophy (SMA) type I is also known as Werdnig–Hoffmann disease. Death usually occurs by 2 to 3 years of age. The progression is rapid and fatal. Patients with SMA type II, known as chronic Werdnig–Hoffmann, usually die by about 10 years of age. Again, the progression is fatal. Patients with SMA type III, known as Kugelberg–Welander disease, have a normal life expectancy with a slow progression.
How well did you know this?
1
Not at all
2
3
4
5
Perfectly
57
Q
NM 10.  Which of the following pharmacologic agents is a first-line treatment for spasticity in patients with amyotrophic lateral sclerosis (ALS)?
       A)  Tizanidine
       B)  Dantrolene
       C)  Benzodiazepine
       D)  Baclofen
A
  1. D) Baclofen is a GABA analogue used to facilitate motor neuron inhibition at spinal levels and is the first-line treatment. Dosing can be started at 5 to 10 mg two to three times per day. It can be titrated up to 20 mg four times per day. Potential side effects include weakness, fatigue, and sedation. Patients must be informed that abrupt discontinuation of baclofen may cause withdrawal seizures. Tizanidine is an alpha-2 agonist. Benzodiazepine can be helpful, but can cause respiratory depression and somnolence. Dantrolene blocks calcium release in the sarcoplasmic reticulum and is ineffective at reducing muscle tone, but can cause generalized muscle weakness.
How well did you know this?
1
Not at all
2
3
4
5
Perfectly
58
Q

NM 11. Which of the following is the most common presenting form of motor neuron disease in adults?
A) Amyotrophic lateral sclerosis (ALS)
B) Poliomyelitis
C) Spinal muscular atrophy (SMA)
D) Primary lateral sclerosis (PLS)

A
  1. A) ALS is also called Lou Gehrig’s disease, as it was named after the New York Yankees’ first baseman who passed away from this disorder. This is unfortunately still the most widely known motor neuron disease and is the most common presenting form. The incidence of ALS is approximately 1.6 to 2.4 cases per 100,000 population.
How well did you know this?
1
Not at all
2
3
4
5
Perfectly
59
Q

NM 12. Which of the following is not included in the Halstead and Rossi (1987) criteria in defining postpolio syndrome?
A) History of previous diagnosis of polio
B) Stability for approximately 5 years
C) Recovery of function
D) No other medical problem to explain new symptoms of weakness/atrophy

A
  1. B) The answer choice would be correct if it was stability for approximately 15 years. Postpolio syndrome is a diagnosis based on exclusion. It has been well defined by the Halstead and Rossi criteria (1987):
  2. Confirmed history of poliomyelitis
  3. Partial to fairly complete neurologic and functional recovery
  4. A period of neurologic and functional stability of at least 15 years in duration
  5. Onset of two or more of the following health problems since achieving a period of stability: unaccustomed fatigue, muscle and/or joint pain, new weakness in muscles previously affected and/or unaffected, functional loss, cold intolerance, new atrophy
  6. No other medical diagnosis to explain these health issues.
How well did you know this?
1
Not at all
2
3
4
5
Perfectly
60
Q
NM 13.  Which virus has been implicated in the development of poliomyelitis?
       A)  Herpes virus
       B)  Papillovirus
       C)  Picornavirus
       D)  Poxvirus
A
  1. C) Acute poliomyelitis is a disease that causes degeneration of the anterior horn cell, and is caused by the polio virus. The polio virus is a small RNA virus belonging to the enterovirus group of the picornavirus family. Picornavirus orally enters the body and spreads via lymphoid system leading to orphaned muscle fibers and potential central nervous system involvement. All other answer choices are DNA viruses.
How well did you know this?
1
Not at all
2
3
4
5
Perfectly
61
Q

NM 43. Which of the following is false regarding multiple sclerosis (MS) and pregnancy?
A) Relapses decrease during pregnancy
B) Higher relapse rate in the first 3 months postpartum
C) Women should not worry about pregnancy worsening their disease process
D) Increased incidence of MS by over 50% among their offspring

A
  1. D) The incidence of MS in the offspring of patients with MS is only slightly increased (3% for girls and 1% for boys). The net effect of pregnancy on the course of MS is neutral. This is due to the fact that there is a decrease in relapse during pregnancy, but higher than normal relapse during the first 3 months postpartum. Therefore, women should not fear that pregnancy can worsen their MS.
How well did you know this?
1
Not at all
2
3
4
5
Perfectly
62
Q

NM 44. Which of the following is true of multiple sclerosis (MS) and pregnancy?
A) Interferon beta is a safe treatment option during pregnancy
B) Glatiramer acetate is a safe treatment option during pregnancy
C) Breastfeeding increases relapse rate in the first 6 months postpartum
D) Restarting disease-modifying agents during breastfeeding is recommended

A
  1. B) Interferon beta is FDA category C for pregnancy and is therefore stopped before a woman decides to try to have a child. Interferon beta has been shown to increase the rate of miscarriage. The risk for relapse is lower overall during pregnancy, so the drug is discontinued before conceiving. Glatiramer acetate is FDA category B and may be a better option for those who want to continue medications during pregnancy. Breastfeeding is known to decrease relapse rate in the first few months postpartum, and it is recommended to restart medications when breastfeeding is stopped.
How well did you know this?
1
Not at all
2
3
4
5
Perfectly
63
Q
NM 45.  The etiology of which disease is thought to be an autoimmune response causing demyelination, axonal damage, and brain atrophy?
       A)  Parkinson’s disease
       B)  Huntington’s disease
       C)  Multiple sclerosis (MS)
       D)  Guillain–Barré syndrome
A
  1. C) MS is considered an autoimmune disease. The disease affects the central nervous system most likely by causing demyelination, leading to plaque formation. The plaque causes oligodendrocyte destruction, astrocyte proliferation, and glial scarring, making the propagation of action potential down a nerve impossible. Remission may occur during this disease process as remyelination occurs.
How well did you know this?
1
Not at all
2
3
4
5
Perfectly
64
Q

NM 46. Which of the following is false regarding multiple sclerosis?
A) Affects males more frequently than females
B) Affects Caucasians more frequently than African Americans
C) Increased incidence in higher socioeconomic class
D) There is no change in long-term relapses in pregnancy

A
  1. A) Multiple sclerosis actually affects females more than males (2:1 female to male ratio). Answer choices B, C, and D are all true. The net effect of pregnancy on the course of MS is neutral. This is due to the fact that there is a decrease in relapse during pregnancy, but higher than normal relapse during the first 3 months postpartum. Therefore, women should not fear that pregnancy can worsen their MS.
How well did you know this?
1
Not at all
2
3
4
5
Perfectly
65
Q
NM 47.  Of the several patterns of multiple sclerosis, which is most common?
       A)  Secondary progressive
       B)  Progressive-relapsing
       C)  Relapsing-remitting
       D)  Primary progressive
A
  1. C) There are six different subtypes of multiple sclerosis (MS). Eighty-five percentage of MS cases are the relapsing-remitting form of MS. This form is characterized by an acute exacerbation followed by a remission period. During the remission, patients can return to their baseline function or may have some form of disability after an exacerbation. The six subtypes of MS are as follows:
  2. Relapsing-remitting
  3. Secondary progressive
  4. Benign
  5. Progressive-relapsing
  6. Primary progressive
  7. Malignant
How well did you know this?
1
Not at all
2
3
4
5
Perfectly
66
Q

NM 48. The following are all considered good prognostic factors in multiple sclerosis (MS) except:
A) Age of onset greater than 35 years
B) Optic neuritis at onset
C) Monosynaptic symptoms
D) Ataxia and tremor

A
  1. D) Ataxia and tremor have a poorer prognosis for MS patients. All others choices are good prognostic factors. Optic neuritis is inflammation of the optic nerve. It may cause sudden, reduced vision in the affected eye and is considered a good prognostic factor when presenting at onset in MS patients.
How well did you know this?
1
Not at all
2
3
4
5
Perfectly
67
Q
NM 49.  The following are common symptoms in patients with multiple sclerosis (MS), except:
       A)  Bowel/bladder dysfunction
       B)  Decreased IQ
       C)  Pain
       D)  Fatigue
A
  1. B) The classic symptoms of MS include bowel/bladder dysfunction, fatigue (which is central in nature), and pain. Other symptoms that patients may report are balance problems, weakness/paralysis, numbness/tingling, spasticity, cognitive problems, depression, emotional lability, and tremor. MS produces a wide variety of problems depending on the location of the lesion in the central nervous system. However, IQ is not usually affected.
How well did you know this?
1
Not at all
2
3
4
5
Perfectly
68
Q
NM 50.  Of the following choices, which is not a major problem affecting activities of daily living (ADLs) reported by multiple sclerosis (MS) patients?
       A)  Sensory disturbance
       B)  Fatigue
       C)  Balance difficulties
       D)  Weakness
A
  1. A) Fatigue is more problematic in patients in the afternoon and can be experienced in 77% of patients with MS. Energy conservation techniques can be used to treat fatigue, and medications such as methylphenidate and amantadine can also be used. Balance difficulties and weakness can contribute to falls in this patient population.
How well did you know this?
1
Not at all
2
3
4
5
Perfectly
69
Q

NM 51. Which of the following can be seen in patients with multiple sclerosis (MS)?
A) Pill rolling tremor
B) Lower motor neuron signs
C) Adson’s sign
D) Weakness and/or Lhermitte’s sign

A
  1. D) Patients commonly present with upper motor neuron (UMN) signs. MS is an autoimmune disease of the central nervous system. UMN signs include hyperreflexia, Hoffmann’s and Babinski’s sign, and spasticity. Weakness and decreased sensation can also be noted. Lhermitte’s sign is a classic finding in MS. Patients complain of an electrical sensation that runs down the back and into the limbs (shoulders). It is elicited by bending the head forward (neck flexion); this is caused by involvement of the posterior columns. There is increased sensitivity of the myelin to stretch or traction. A pill-rolling tremor is frequently seen in Parkinson’s disease.
How well did you know this?
1
Not at all
2
3
4
5
Perfectly
70
Q

NM 52. The pathognomonic test for multiple sclerosis (MS) includes:
A) Increased cerebrospinal fluid (CSF) protein, oligoclonal IgG bands
B) Multifocal “bright” areas of hyperintensity on T2-weighted images
C) Both of the above
D) None of the above

A
  1. D) Although both answer choices A and B may be seen in patients with MS, there is no pathognomonic test for MS. These tests are nonspecific and have to be interpreted with the entire clinical picture.
How well did you know this?
1
Not at all
2
3
4
5
Perfectly
71
Q

NM 53. Corticosteroid use in multiple sclerosis (MS) is considered:
A) Effective in long-term use
B) Most responsive in cerebellar and sensory symptoms
C) To speed recovery
D) To prevent further attacks

A
  1. C) Corticosteroids (methylprednisolone) are used in acute attacks in MS patients. Corticosteroids have anti-inflammatory and antiedema properties. The dose that is given is 500 to 1,000 mg/day IV for 3 to 5 days and can be given with or without an oral taper. Long-term use is not recommended, as hyperglycemia, hypertension, osteoporosis, and cataracts can occur with prolonged use. Cerebellar and sensory symptoms are least responsive to corticosteroids. Steroids can hasten recovery, but do not prevent further attacks, or alter progression.
How well did you know this?
1
Not at all
2
3
4
5
Perfectly
72
Q
NM 54.  Which first-line treatment choice can reduce relapse in multiple sclerosis (MS) patients?
       A)  Corticosteroids
       B)  Immunomodulator agents
       C)  Intravenous immunoglobulin
       D)  Immunosuppressive agents
A
  1. B) Immunomodulator agents include interferon beta-1a (Avonex and Rebif), interferon beta-1b (Betaseron), and glatiramer acetate (Copaxone). These have been shown to reduce relapse rate in MS patients. Immunosuppressive agents include cyclosporin, azathioprine, methotrexate, and mitoxantrone and can reduce relapse rate, but have a greater side-effect profile and are therefore used as a second-line agent. Intravenous immunoglobulin is also an immunosuppressive agent and is still being studied for use in this patient population. Steroids can hasten recovery, but do not prevent further attacks, or alter progression.
How well did you know this?
1
Not at all
2
3
4
5
Perfectly
73
Q

NM 59. Neuroplasticity is a concept that refers to all of the following except:
A) The potential ability of the central nervous system (CNS) to modify its structural and functional organization
B) Partial recovery is possible long after sustaining a brain injury
C) The brain remains capable of changing in response to experience and injury
D) Insult or injury to the CNS is permanent and functional ability cannot be altered with any type of intervention

A
  1. D) Insult or injury to the CNS is permanent and functional ability cannot be altered with any type of intervention. Neuroplasticity refers to the dynamic nature of the brain and CNS and its ability to change in response to experience and injury.
How well did you know this?
1
Not at all
2
3
4
5
Perfectly
74
Q

NM 60. Which of the following statements regarding constraint-induced movement therapy (CIMT) is true?
A) It requires constraint of the affected extremity
B) It is based on principles of repeated practice and intense activity
C) It utilizes a passive nonintensive approach
D) It aims to increase the use of the unaffected extremity

A
  1. B) CIMT is an intervention directed at improving the function of the affected upper extremity after a brain injury. It involves intensive motor training of up to 6 hours daily and motor restriction (constraint) of the unaffected extremity. CIMT is based on research findings that the affected limb is negatively impacted by learned nonuse because of increased dependence on the intact limb.
How well did you know this?
1
Not at all
2
3
4
5
Perfectly
75
Q
NM 61.  Neural strategies of functional improvement after central nervous system injury include all of the following except:
       A)  Restoration
       B)  Redacting
    C)  Recruitment
       D)  Retraining
A
  1. B) Restoration focuses on reengaging residual brain areas that are initially dysfunctional after an injury. Recruitment involves engaging new residual areas of the brain. Retraining involves training the residual brain to perform new functions. Redacting refers to editing or drafting a document for publication
How well did you know this?
1
Not at all
2
3
4
5
Perfectly
76
Q

NM 62. Benefits of partial body weight supported gait training include:
A) Addresses ambulation issues for patients who have sufficient strength and balance
B) Enhances development of compensatory gait strategies
C) Provides earlier weight-bearing to increase strength and increase spasticity
D) Allows for the simulation of task-specific walking movements

A

D) Partial body weight supported gait training allows the simulation of task-specific walking movements and enables therapists to assist patients in the components of gait rather than bearing body weight. Gait training is necessary for the restoration of self-ambulation after brain injury. The patient works at improving coordination of movement and gradually increasing muscle strength. The lack of sufficient strength and balance to maintain an erect posture typically prevents gait training. Partial body weight support gait training is postulated to result in earlier weight-bearing, increased strength, and reduced spasticity. It helps to prevent development of compensatory strategies for ambulation that may develop while using a cane or walker and creates undesirable motor habits. It also reduces the demands of muscles.

How well did you know this?
1
Not at all
2
3
4
5
Perfectly
77
Q
NM 63.  Neuroplasticity is not positively influenced by:
       A)  Environment and stimulation
       B)  Repetition of tasks
       C)  Motivation
       D)  Compensation
A
  1. D) Environmental manipulation might influence both morphological change and the functional outcome after a cortical injury. Research studies revealed that animals reared or housed as adults in complex environments led to enhanced dendritic growth that was correlated with enhanced behavioral abilities. Patient motivation is an important factor in the success of his or her rehabilitation program. An important component of the rehabilitation process is to assist the patient in avoiding development of negative compensatory strategies.
How well did you know this?
1
Not at all
2
3
4
5
Perfectly
78
Q

NM 66. Which of the following features is considered a good prognosis indicator in patients with multiple sclerosis?
A) Age at onset greater than 35 years
B) Rapidly progressive onset
C) Sensory findings/optic neuritis at onset
D) Male sex

A
  1. C) All of the above features portend a poor prognosis except for C, sensory findings/optic neuritis at onset. Other good prognostic indicators include a sudden onset with a long remission, retained ability to ambulate, and low current disability.
How well did you know this?
1
Not at all
2
3
4
5
Perfectly
79
Q

NM 67. All of the following symptoms are seen in multiple sclerosis (MS) except:
A) Impairment of deep sensation, proprioception
B) Scanning speech
C) Impaired convergence
D) Bowel and bladder incontinence

A
  1. C) All the above are symptoms of MS except for choice C. Choice A is due to demyelination of the dorsal column tracts. Choice B is seen in “Charcot’s triad” or advance stages of the disease where scanning speech, intention tremor, and nystagmus can manifest themselves. Scanning speech is characterized as being explosive speech that is broken into syllables. Bowel and bladder incontinence are symptoms seen due to demyelination of the spinal cord. Convergence is one of the optical features not seen in MS, whereas nystagmus, optic neuritis, diplopia, and internuclear ophthalmoplegia (a disorder of lateral conjugate gaze in which the affected eye has impaired adduction, causing diplopia) may be noted. However, convergence is generally intac
How well did you know this?
1
Not at all
2
3
4
5
Perfectly
80
Q

NM 68. Lhermitte’s sign is produced by:
A) Passive neck flexion causing electrical shock-like sensation radiating to the spine, shoulders, and other areas
B) Axial load placed by pressing downward force on top of the patient’s head causing reproducible numbness to one or both limbs
C) Rotating head to side with neck extended with ipsilateral shoulder abducted at 45 degrees and elbow extended with patient inhaling and holding inspiration
D) None of the above

A
  1. A) Lhermitte’s sign is commonly seen in multiple sclerosis (MS), but is not universally seen in all cases. This test assesses for increased sensitivity of the myelin to traction and is similar to an upper motor neuron sign. Choice B refers to Spurling’s test, which is done to test for radiculopathies. Choice C is Adson’s maneuver, which is done to assess for neurogenic thoracic outlet syndrome by causing scalene compression of the brachial plexus.
How well did you know this?
1
Not at all
2
3
4
5
Perfectly
81
Q

NM 69. Although a multitude of tests can be done to help diagnose multiple sclerosis (MS), which of the following is not suggestive of this diagnosis?
A) Multifocal bright T2-weighted periventricular images
B) Increased latency seen in somatosensory evoked potentials (SSEP)
C) Increased cerebrospinal fluid (CSF) protein with oligoclonal bands
D) Decreased amplitudes of sensory nerve action potentials (SNAPs) and compound motor action potentials (CMAPs) in nerve conduction studies (NCSs)

A
  1. D) All the following are suggestive findings of MS except for choice D. Often, clinical evidence alone is diagnostic of MS if there is an apparent history of repeated attacks and signs of more than two lesions. However, in other cases, magnetic resonance imaging (MRI) findings and spinal tap can help assist in the diagnosis. MRI has the greatest sensitivity, as lesions of white matter are seen in approximately 85% of the cases involving the optic nerves, spinal cord, and brain. Enhancement with gadolinium is a sensitive indicator of disease activity. CSF tap will show increased protein because of the lost myelin and increased amount of IgG. SSEPs are frequently delayed, since MS is a central process and generally involves the cord. Since NCSs test peripheral nerves, they are generally not affected.
How well did you know this?
1
Not at all
2
3
4
5
Perfectly
82
Q
NM  70.  Which of the following is not used in the treatment of multiple sclerosis (MS)?
       A)  Corticosteroids
       B)  Interferon beta
       C)  Glatiramer acetate
       D)  Rituximab
A
  1. D) All the above are used in the treatment of MS except for choice D, which is used in Devic’s disease or neuromyelitis optica. Corticosteroids are often used in acute attacks and are used in high doses and tapered down. Interferon beta and glatiramer acetate are immunomodulating agents that help in decreasing the progression of the disease. Rituximab is a monoclonal antibiotic against B-cell CD20 antigen used in hematological diseases (lymphomas) and many autoimmune diseases, but not used in MS.
How well did you know this?
1
Not at all
2
3
4
5
Perfectly
83
Q

NM 71. What is Uhthoff’s phenomenon?
A) Passive neck flexion causing shock-like symptoms radiating to the spine and shoulders
B) Inability to adduct the left eye on right lateral gaze and inability to adduct the right eye on left lateral gaze with intact convergence
C) Worsening of neurological symptoms including visual problems seen with increased body temperature
D) None of the above

A
  1. C) Choice A refers to Lhermitte’s sign, and choice B refers to internuclear ophthalmoplegia (INO) all seen in multiple sclerosis. Choice C describes Uhthoff’s phenomenon. This effect is thought to reflect areas of impaired but still functioning myelin that breaks down in transmitting electrical impulses when surrounding fluid is heated.
How well did you know this?
1
Not at all
2
3
4
5
Perfectly
84
Q
NM 72.  What are the typical features of amyotrophic lateral sclerosis (ALS) associated with lower motor neuron disease?
       A)  Babinski sign
       B)  Increased tone
       C)  Spasticity
       D)  Fasciculations
A
  1. D) Increased tone, Babinski sign, and spasticity are all signs of an upper motor neuron (UMN) syndrome, whereas fasciculations are a sign of lower motor neuron syndrome. ALS is a disease with progressive injury and death to both pyramidal (upper motor) neurons and anterior horn (lower motor) neurons.
How well did you know this?
1
Not at all
2
3
4
5
Perfectly
85
Q
NM 75.  Which of the following medications used for spasticity has the least amount of sedation and cognitive impairment?
       A)  Diazepam
       B)  Dantrolene sodium
       C)  Baclofen
       D)  Clonidine
A
  1. B) Although seldom used for spasticity, dantrolene has the least amount of sedation and cognitive impairment of all the antispasticity medications. This is primarily because its mechanism of action takes place peripherally in the sarcoplasmic reticulum, decreasing calcium release. It is an agent that is considered in the treatment of spasticity for patients suffering from brain injury.
How well did you know this?
1
Not at all
2
3
4
5
Perfectly
86
Q
NM 77.  Arnold–Chiari malformation type II (the downward displacement of the medulla and brainstem through the foramen magnum causing kinking of the brainstem) is associated with which condition?
       A)  Spina bifida occulta
       B)  Meningocele
       C)  Myelomeningocele
       D)  All the above
A
  1. C) In spina bifida, a congenital malformation of the vertebral columns and spinal cord exists. Only in myelomeningocele, where the protruding sac contains meninges, spinal cord, and spinal fluid, is there an associated finding of Arnold–Chiari malformation, which presents in approximately 80% to 90% of patients.
How well did you know this?
1
Not at all
2
3
4
5
Perfectly
87
Q

NM 78. Which of the following is not true regarding spina bifida?
A) Complicated by hydrocephalus in 90% of cases
B) Of those with Arnold–Chiari malformation, more than 80% of children will require ventriculoperitoneal shunting
C) Three-fold increase in the incidence of patients suffering from lower IQ than the normal population
D) Most common cause of death is cardiovascular complications

A
  1. D) All of the above statements are correct except for D. The number one cause of death in spina bifida is central respiratory dysfunction.
How well did you know this?
1
Not at all
2
3
4
5
Perfectly
88
Q

NM 86. In the Miller Fischer variant (MFV) of Guillain–Barré syndrome (GBS), what are the classical findings?
A) Ophthalmoplegia, ataxia, and areflexia
B) Dysrhythmias, impaired diaphoresis, photophobia
C) Dysphagia, nausea, diarrhea
D) All of the above

A
  1. A) The classic findings in the MFV of GBS are ophthalmoplegia, ataxia, and areflexia. MFV represents only 5% of GBS cases and presents in descending fashion as opposed to the conventional ascending paralysis seen in conventional GBS forms. Thus, the eye is usually affected first. Gait and trunk muscles are often affected with general sparing of the limb muscles. The other choices B and C are seen in another variant of GBS, acute pan-autonomic neuropathy (which is the most rare form and quite fatal).
How well did you know this?
1
Not at all
2
3
4
5
Perfectly
89
Q
NM 100. Amyotrophic lateral sclerosis (ALS) is a lesion of:
       A)  Upper motor neurons
       B)  Lower motor neurons
       C)  Upper and lower motor neuron
       D)  None of the above
A
  1. C) In ALS, there is weakness and muscle atrophy caused by degeneration of upper and lower motor neurons.
How well did you know this?
1
Not at all
2
3
4
5
Perfectly
90
Q
NM 101. Lower motor neuron (LMN) signs include all of the following except:
       A)  Spasticity
       B)  Atrophy
       C)  Hyporeflexia
       D)  Fasciculations
A
  1. A) LMN lesions can present with flaccidity, atrophy, and fasciculations. Spasticity is a sign of an upper motor neuron lesion.
How well did you know this?
1
Not at all
2
3
4
5
Perfectly
91
Q
NM 102. In a patient with multiple sclerosis (MS), pregnancy usually results in:
       A)  Death
       B)  Symptom relapse
       C)  Complete recovery
       D)  Decreased relapses
A
  1. D) Pregnancy in MS decreases relapses. However, relapses may increase after the delivery.
How well did you know this?
1
Not at all
2
3
4
5
Perfectly
92
Q

NM 103. Lhermitte’s sign is classically seen, but is not pathognomonic, for this disease:
A) Multiple sclerosis (MS)
B) Amyotrophic lateral sclerosis (ALS)
C) Duchenne muscular dystrophy
D) Poliomyelitis

A
  1. A) Lhermitte’s sign is most commonly seen in MS. The patient may experience an electric shock–like sensation radiating to the spine and extremities when the neck is flexed. This is thought to be due to increased myelin sensitivity to traction.
How well did you know this?
1
Not at all
2
3
4
5
Perfectly
93
Q
NM 104. Fatigue in a patient with multiple sclerosis (MS) will increase with:
       A)  Heating
       B)  Cooling
       C)  Wetness
       D)  Dryness
A
  1. A) Heat can worsen symptoms in patients with MS. They should be encouraged to use air conditioning. If pool therapy is used, the water should be below 84°F (29°C). When exercising, these patients must avoid raising their body core temperature.
How well did you know this?
1
Not at all
2
3
4
5
Perfectly
94
Q
NM 107. A baby with spinal muscular atrophy (SMA) type I (Werdnig–Hoffmann disease) may present with:
       A)  Weak cry
       B)  Dysphagia
       C)  Weak suck
       D)  All of the above
A
  1. D) SMA is an autosomal recessive disorder of infancy that presents within the first 2 months of life with hypotonia and symmetric weakness of the lower extremities more than the upper extremities. This is due to degeneration of the anterior horn cell of the spinal cord and brainstem. All of the above are presenting symptoms of SMA type I, which is the most severe. Death usually occurs by age 3.
How well did you know this?
1
Not at all
2
3
4
5
Perfectly
95
Q

NM 117. Which of the following can trigger Guillain–Barré syndrome (GBS)?
A) Respiratory viral infection
B) Gastrointestinal viral infection
C) Vaccinations
D) All of the above

A
  1. D) GBS is believed to be caused by Campylobacter jejuni or a viral attack on the myelin and Schwann cells. Initial symptoms will include weakness and tingling sensation in the legs and can progress to almost total paralysis.
96
Q
NM 118. Guillain–Barré syndrome (GBS) may be treated with all of the following except:
       A)  Rehabilitation
       B)  Plasmapheresis
       C)  Steroids
       D)  IV immunoglobulins (IVIgs)
A
  1. C) Steroids have not proven to be an effective treatment for GBS. IVIg is usually started first owing to the ease of administration and safety profile. Plasmapheresis hastens recovery when used in the first 4 weeks of symptom onset. Rehabilitation usually follows the acute phase treatment modalities
97
Q

AN 27. Which ligament of the spine resists extension?
A) Posterior longitudinal ligament
B) Ligamentum flavum and facet joint capsule
C) Anterior longitudinal ligament
D) Interspinous and supraspinous ligaments

A
  1. C) The anterior longitudinal ligament runs anteriorly and vertically attaching to the front of each vertebra. It traverses all of the vertebral bodies and intervertebral discs.
98
Q
AN 55.  The Achilles reflex assesses which nerve root?
       A)  L4
       B)  L5
       C)  S1
       D)  S3
A
  1. C) The ankle jerk (Achilles reflex), occurs when the Achilles tendon is tapped when the foot is slightly dorsiflexed and produces plantar flexion.
99
Q

PO 69. A patient with a C6 level spinal cord injury would find this splint useful to help pick small objects up using a three jaw chuck pinch:
A) Kleinert splint
B) Adaptive usage device splint
C) Dynamic splint
D) Rehabilitation Institute of Chicago tenodesis splint

A
  1. D) This splint uses cords running from the wrist across the palm and attach between the index and the ring fingers. The cords are lax when the wrist is released and tighten when the wrist is extended helping bring the fingers closer to the immobilized thumb.
100
Q
PO 75.  Which hand orthosis employs wrist extension for closing the hand?
       A)  Wrist hand orthosis
       B)  Wrist orthosis
       C)  Tenodesis orthosis
       D)  Kleinert orthosis
A
  1. C) Tenodesis splints are a special class of orthosis prescribed mainly in tetraplegic patients. They employ wrist extension to tighten the finger flexor tendons causing the hand to close.
101
Q
PED 30.  The most common site of spinal cord injury (SCI) in children is:
       A)  Thoracic
       B)  Lumbar
       C)  Cervical
       D)  Sacral
A

C) The most mobile portion of the spinal column is the cervical region (hence the most vulnerable). Approximately 55% of SCI in children involve the cervical spine, 30% the thoracic spine, and 15% the lumbar spine.

102
Q
PED 31.  Which condition can predispose children to cervical spinal cord injury because of atlantoaxial dislocation?
       A)  Down syndrome
       B)  Klippel–Feil syndrome
       C)  Morquios syndrome
       D)  Achondroplasia
       E)  All of the above
A
  1. E) The odontoid process helps to prevent dislocation of C1 onto C2. Aplasia of the odontoid process can occur in mucopolysaccharidosis (Morquios syndrome) or Klippel–Feil syndrome. In Down syndrome, congenital hypoplasia of the articulation of C1 and C2 can occur.
103
Q
PED 32.  Spinal cord injury without radiographic abnormalities (SCIWORA) is most common in:
       A)  Infants and young children
       B)  Adolescents
       C)  Adults
       D)  Old age geriatric population
A
  1. A) SCIWORA is most common in children below 8 years of age. The vertebral column in this population has more elasticity, hence fracture and dislocation are less common. However, myelopathy or central cord injury can occur. MRI of the spine is more sensitive than CT or radiography to reveal SCIWORA injuries.
  2. A) In anterior spinal
104
Q

PED 33. A 16-year-old football player suffers a T10 fracture. He is paraparetic and has no sensation below T10. However, his proprioception, light touch, and vibration are intact.
These findings are suggestive of:
A) Anterior spinal cord syndrome
B) Posterior spinal cord syndrome
C) Brown-Séquard syndrome
D) Conus medullaris syndrome

A
  1. A) In anterior spinal cord syndrome, there is paresis and analgesia below the lesion, but vibration and position sense are preserved as these are mediated by the posterior columns. In Brown-Séquard syndrome, there is ipsilateral motor paralysis, loss of touch as well as proprioception, and contralateral loss of pain and temperature below the level of the lesion.
105
Q
PED 34.  In adults, at what level does the spinal cord terminate?
       A)  L1
       B)  L3
       C)  L4
       D)  T10
A
  1. A) The spinal cord terminates at L1 in adults and L2 in infants.
106
Q

PED 35. Spinal cord injuries are classified by which of the following scales?
A) Glasgow Coma Scale
B) American Spinal Injury Association/International Medical Society of Paraplegia (ASIA/IMSOP) spinal cord impairment scale
C) The Wee Functional Independence Measure Scale
D) Barthel index

A
  1. B) ASIA/IMSOP (American Spinal Injury Association/International Medical Society of Paraplegia) utilizes documentation of motor, sensory, and sphincter functions. Ten key muscles are graded using the Medical Research Council (MRC) scale, and sensation is assessed over 28 dermatomes.
    The Glasgow Coma Scale is used in traumatic brain injury.
    The Wee functional independence score measures self-care, mobility, cognition, and communication in children.
    The Barthel index is used to measure performance in activities of daily living following conditions such as stroke.
107
Q
PED 36.  Phrenic nerve pacing may be required with spinal cord injury at which level?
       A)  C3–5
       B)  T3–5
    C)  T7–8
       D)  T12–4
A
  1. A) The phrenic nerve is formed from nerve roots C3–5 and supplies the diaphragm. Patients who require permanent ventilatory support can be candidates for phrenic nerve pacing.
108
Q

PED 38. In which type of spinal muscular atrophy (SMA) can most of the patients sit but not walk?
A) Type I
B) Type II
C) Type III

A
  1. B) SMA type I (Werdnig–Hoffmann disease) has its onset before 6 months of age. Most patients cannot sit, and the condition has a mortality rate of more than 90% by 3 months of age.
    SMA type II has an onset between 6 and 18 months of age. Most patients can sit but not walk. Life expectancy in these patients is to the second decade.
    SMA type III (Kugelberg–Welander syndrome) usually presents at about 18 months of age. Most patients can walk and have a normal life expectancy.
109
Q

PED 39. Most children with spinal muscular atrophy (SMA) have:
A) Mental retardation
B) A single crease across their palm
C) Epilepsy
D) High cognitive function

A
  1. D) SMA is a disease affecting the anterior horn cells (motor neurons) that causes neurogenic atrophy of muscles. The brain is not affected. They have high cognitive abilities. Therefore, providing them with tools for ambulation and mobility are important. Patients with Down syndrome usually have a single crease across their palms instead of two creases.
110
Q
PED 40.  All of the following are anterior horn cell disease affecting the motor neuron except:
       A)  Spinal muscular atrophy
       B)  Botulism
       C)  Amyotrophic lateral sclerosis
       D)  Poliomyelitis
A
40.  All of the following are anterior horn cell disease affecting the motor neuron except:
       A)  Spinal muscular atrophy
       B)  Botulism
       C)  Amyotrophic lateral sclerosis
       D)  Poliomyelitis
111
Q

PED 41. Lesions of the facial nerve distal to the nucleus (lower motor neuron involvement) result in:
A) Paralysis of the lower facial muscles
B) Paralysis of the upper (forehead) and lower facial muscles
C) Paralysis of the lower facial muscles with sparing of upper facial muscles (forehead)
D) Paralysis of upper and lower facial muscles as well as the muscles of mastication

A
  1. B) When forehead muscles are involved in facial nerve palsy, it is indicative of a lower motor neuron lesion distal to the nucleus. The muscles of mastication are supplied by cranial nerve V. Lower facial muscles are involved in both upper and lower motor neuron facial palsies. Bilateral upper motor neuron innervation of the upper facial muscles leads to sparing of these muscles in an upper motor neuron lesion.
112
Q
PED 42.  In Erb’s palsy, all of the following muscles are paralyzed except:
       A)  Deltoid
       B)  Bicep brachialis
       C)  Supinator
       D)  Intrinsic muscles of hand
A

. D) Erb’s palsy involves injury to the upper trunk of the brachial plexus. Intrinsic muscles of hand are supplied by the ulnar nerve, which is supplied by C8-T1. In Erb’s palsy, the arm is held in adduction and internal rotation at the shoulder, extension at the elbow, pronation of the forearm, and flexion at the wrist.

113
Q
PED 43.  The predominant type of neuropathy in Guillain–Barré syndrome (acute inflammatory demyelinating polyradiculoneuropathy) is:
       A)  Motor
       B)  Sensory
       C)  Ataxia
       D)  Autonomic
A
  1. A) Although all of the above can be seen, motor involvement predominates. Campylobacter jejuni, Mycoplasma pneumoniae, cytomegalovirus, and Epstein–Barr virus are common known causes.
114
Q

PED 52. You are providing rehabilitation to a 12-year-old spinal cord injured boy. He has been complaining of intermittent headaches with nasal congestion and nausea. He perspires profusely on his face and neck. His blood pressure is elevated intermittently. The most likely level of his spinal cord injury is:
A) T6
B) T12
C) L4

A
  1. A) This patient has an acute and potentially life-threatening syndrome called autonomic dysreflexia. This can occur with complete spinal cord injuries at the T6 level or above and is due to imbalanced sympathetic discharges of the splanchnic outflow.
115
Q
PED 80.  Arnold–Chiari malformation complicated by hydrocephalus occurs most commonly in which type of spina bifida?
       A)  Spina bifida occulta
       B)  Meningocele
       C)  Myelomeningocele
       D)  Myelocele
A
  1. C) This malformation is seen in more than 90% of the cases with myelomeningocele. The protruding sac contains meninges, spinal cord, and spinal fluid. Spina bifida occulta results when there is failure of fusion of the posterior elements of the vertebrae. Meningocele is characterized by a protruding sac, which contains meninges and spinal fluid, whereas myelocele is the presence of a cystic cavity in front of the anterior wall of the spinal cord.
116
Q

PED 81. Which type of spinal muscular atrophy (SMA) is associated with good long-term survival?
A) Type I (Werdnig–Hoffmann disease)
B) Type II
C) Type III (Kugelberg–Welander syndrome)
D) Type IV

A

PED 81. C) Type III, or Kugelberg–Welander syndrome, is associated with good long-term survival but dependent on respiratory function. It is an autosomal recessive disorder characterized by proximal weakness predominantly of the legs. Fasciculations are common, and scoliosis is frequent

117
Q

PED 113. Intrathecal baclofen is a treatment that is recommended for:
A) A child with mixed tone abnormalities who has failed to respond adequately to oral medications
B) A child with mild spastic diplegia
C) A child with mental retardation, seizure disorder, and orthopedic deformity
D) A child with severe spasticity and parents with a known history of noncompliance

A
  1. A) Intrathecal baclofen should be considered for a child with significant spasticity or dystonia interfering with function and who has had an adequate trial of oral medications.
118
Q

PED 115. When advising families with children with chronic illness and disability, practitioners should:
A) Remain dogmatic on traditional therapies alone
B) Allow for open discussion of benefits only
C) Allow for a risk-benefit discussion
D) Understand that families are angry and need to act out with defiance

A
  1. C) According to the American Academy of Pediatrics policy statement on complementary and alternative medicine, families that request complementary treatments should engage in an open, risk-benefit discussion.
119
Q
PED 116. What percentage of the population age 1 to 10 has spina bifida occulta?
       A)  1%
       B)  8%
       C)  17%
       D)  30%
A
  1. C) Studies show a 17% incidence of spina bifida occulta in normal individuals age 1 to 10, with no neurologic involvement and incomplete closure of the posterior elements of the spine.
120
Q
PED 117. What is the incidence of myelomeningocele (MMC) in the United States?
       A)  2 per 1,000 live births
       B)  20 per 1,000 live births
       C)  100 per 1,000 live births
       D)  10 per 1,000 live births
A
  1. A) Rates have varied in the United States from 2.34 per 1,000 live births to as low as 0.51 per 1,000 live births. The declining rate could be related to awareness of the need for dietary supplements during pregnancy, especially folic acid.
121
Q
PED 118. Which of the following interventions was found to reduce the frequency of myelomeningocele (MMC)?
       A)  Vitamin B
       B)  Vitamin E
       C)  Folic acid
       D)  Ascorbic acid
A
  1. C) Folic acid supplementation was first shown to decrease the rate of neural tube deficits in the early 1980s in Wales
122
Q
PED 119. What percentage of myelomeningoceles (MMCs) are associated with hydrocephalus at birth?
       A)  5%
       B)  15%
       C)  25%
       D)  50%
A
  1. B) Approximately 15% of these babies have severe hydrocephalus and require immediate shunting.
123
Q

PED 120. Which is/are the most common shunt complication(s) when treating hydrocephalus in an infant with myelomeningocele?
A) Infection and obstruction
B) Lower limb weakness
C) Pain and swelling
D) Incontinence of bowel and bladder

A
  1. A) The two most common shunt complications are infection and obstruction. Signs and symptoms vary with the age of the child.
124
Q

PED 121. What is a leading cause of death for infants with myelomeningocele (MMC)?
A) Symptomatic Chiari II malformation
B) Infection
C) Hydrocephalus
D) Renal failure

A
  1. A) Symptomatic Chiari II malformation remains the leading cause of death for infants with MMC.
125
Q
PED 122. What are frequent signs and symptoms of spinal cord “tethering”?
       A)  Infection
       B)  Increased weakness
       C)  Cognition loss
       D)  Gastrointestinal upset
A
  1. B) Cord tethering may result in increasing weakness, scoliosis, pain, urologic dysfunction, or orthopedic deformities.
126
Q
PED 123. What percentage of those with myelomeningocele (MMC) will have normal urinary control?
       A)  50%
       B)  25%
       C)  10%
       D)  1%
A
  1. C) Fewer than 10% of children with MMC have normal urinary control. Continence is an important issue and must be addressed.
127
Q
PED 124. What is a frequent allergy in myelomeningocele (MMC)?
       A)  Soap
       B)  Poison ivy
       C)  Latex
       D)  Urine
A
  1. C) Although the prevalence of latex allergy in the general population is estimated to be less than 1% to 2%, its prevalence in children with MMC ranges from 20% to 65% because of the repeated exposure.
128
Q
PED 125. What is the most common level of myelomeningocele (MMC)?
       A)  Upper thoracic
       B)  Cervical
       C)  Lower thoracic
       D)  Lumbar
A
  1. D) The majority of children with MMC have lumbar lesions, with one-fourth having midlumbar lesions. Very few have cervical and upper thoracic levels.
129
Q

PED 126. Which is the major cause of calcaneal deformity in myelomeningocele (MMC)?
A) Unopposed contraction of foot dorsiflexors
B) Unopposed contraction of plantar flexors
C) Weakness of foot intrinsics
D) Hip dislocation

A
  1. A) Calcaneal deformities result from unopposed contraction of foot dorsiflexors and can be present at birth or develop later
130
Q
PED 127. Which would best describe the ambulation potential for a patient with thoracic myelomeningocele (MMC)?
       A)  Community ambulatory
       B)  Household ambulatory
       C)  Functional ambulatory
       D)  Nonambulatory
A
  1. C) Most patients with lumbar lesions will have some level of ambulation, but those with thoracic MMC can be functional ambulators.
131
Q

PED 128. Which of the following affects IQ scores of children with myelomeningocele (MMC)?
A) Central nervous system infections
B) Recurrent shunt revisions
C) Ability to ambulate
D) Pressure ulcers

A
  1. A) IQ scores are adversely affected by central nervous system infections but not by recurrent shunt revisions.
132
Q

PED 129. Which best describes the development of female patients with myelomeningocele (MMC)?
A) Normal
B) Markedly reduced
C) Increased likelihood of late puberty
D) Increased likelihood of precocious puberty

A
  1. D) Between 12% and 15% of girls with MMC show precocious puberty; 95% have menses.
133
Q

PED 130. Of those patients with myelomeningocele (MMC) who complete high school, about what percentage go on to further education?
A)

A
  1. D) About one-half of those who finish high school go on to further education
134
Q

PED 131. What must accompany rehabilitation in those with myelomeningocele (MMC) as they move to adulthood?
A) Anticipation and prevention of life-threatening events
B) Rechecks as needed
C) Avoidance of activity
D) Insistence on employment

A
  1. A) MMC presents lifelong challenges to affected patients, their families, and clinicians. Surveillance and education are required to prevent life-threatening events related to ventriculoperitoneal shunt malfunction, Chiari II malformation, renal failure, infection, and latex allergy-all in conjunction with maximizing function.
135
Q
MED 129. Spinal cord injury (SCI) in a cancer patient occurs most commonly from:
       A)  Spinal cord tumors
       B)  Spinal metastases
       C)  Radiation
       D)  Chemotherapy
A
  1. B) Neoplastic cancer related spinal cord compression comprises up to 14% of new onset SCI. The vast majority is metastatic (up to 85% of the cases). Spinal metastasis is seen in 15% to 40% of all systemic cancers. The primary cancer site for spinal metastasis is most commonly breast, lung, and prostate cancer.
136
Q
MED 63.  Among the following, which represents primary respiratory muscle(s) during quiet respiration?
       A)  Diaphragm
       B)  External intercostal muscles
       C)  Abdominal muscles
       D)  Trapezius
A
  1. A) During quiet respiration, the primary muscle of respiration is the diaphragm. During exercise, additional muscles become involved (such as the external intercostals during inspiration, the abdominal muscles, and the scalenus).
137
Q

MED 64. What is the term for the volume of gas in the lungs at maximal inspiration?
A) Functional residual capacity (FRC)
B) Vital capacity (VC)
C) Residual volume (RV)
D) Total lung capacity (TLC)

A
  1. D) TLC is achieved after maximal effort of the muscles of inspiration to expand the lungs. The FRC is the volume of air remaining at the end of a normal exhalation. The RV is the volume remaining in the lungs after maximal exhalation. VC represents the volume of air that can be exhaled after maximal inspiration.
138
Q

MED 65. The volume of gas in the lungs at the end of normal expiration is called:
A) Functional residual capacity (FRC)
B) Vital capacity (VC)
C) Residual volume (RV)
D) Expiratory reserve volume (ERV)

A
  1. A) The FRC is the volume of air remaining at the end of a normal exhalation. VC represents the volume of air that can be exhaled after maximal inspiration. The RV is the volume remaining in the lungs after maximal exhalation. The ERV is the amount of air that can be exhaled after a normal expiration.
139
Q
MED 66.  The amount of gas moving in and out of the lungs during resting respiration is called:
       A)  Vital capacity (VC)
       B)  Total lung capacity (TLC)
       C)  Tidal volume
       D)  Forced vital capacity (FVC
A
  1. C) Tidal volume is the volume of air moved in and out of the lungs during normal respiration. VC represents the volume of air that can be exhaled following maximal inspiration. TLC is achieved following maximal effort of the muscles of inspiration to expand the lungs. The FVC is the total volume of air expired after a full inspiration.
140
Q
MED 68.  What nerve supplies the diaphragm?
       A)  Long thoracic nerve
       B)  Thoracodorsal nerve
       C)  Lateral pectoral nerve
       D)  Phrenic nerve
A
  1. D) The diaphragm is supplied by the phrenic nerve (C3-C5 nerve roots). The long thoracic nerve (C5-C7) supplies serratus anterior. The thoracodorsal nerve (C7-C8) supplies latissimus dorsi, and the lateral pectoral nerve (C5-C7) supplies the pectoralis major muscle.
141
Q
MED 71.  Which nerve roots contribute to the phrenic nerve?
       A)  C1-C3
       B)  C5-C7
       C)  C3-C5
       D)  C7-T1
A
  1. C) The phrenic nerve originates from the C3, C4, and C5 nerve roots.
142
Q

MED 78. Which device aids in secretion clearance by applying a positive pressure to the airways followed by a negative pressure?
A) Mechanical insufflator-exsufflator
B) Yankauer suction wall unit
C) Bilevel positive airway pressure (BiPAP)
D) Chest percussion

A
  1. A) A mechanical insufflator-exsufflator (e.g., Cough-Assist) is a device that aids in the clearance of bronchopulmonary secretions in patients who have conditions such as neuromuscular disease, traumatic brain injury, cerebral palsy, and tetraplegia. This machine delivers deep inspiration through a face mask or adapter connecting to the patient’s tracheostomy tube, followed immediately by a suction pressure (negative pressure). This device can be adjusted so that at least 2 to 4 seconds are allotted to allow for maximal chest expansion before rapid lung emptying.
143
Q
MED 79.  When would a mechanical insufflator-exsufflator be contraindicated?
       A)  High spinal cord injury
       B)  Bullous emphysema
       C)  Cerebral palsy
       D)  Neuromuscular disease
A
  1. B) Mechanical insufflator-exsufflator is indicated for secretion management in conditions such as neuromuscular disease, high cervical spinal cord injuries, traumatic brain injuries, and cerebral palsy where patients may have
144
Q

MED 84. Which is a reasonable treatment modality for moderate to severe obstructive sleep apnea (OSA)?
A) Tracheal intubation
B) Diaphragmatic pacing
C) Nocturnal pulse oximetry
D) Continuous positive airway pressure (CPAP)

A
  1. D) Nasal CPAP (n-CPAP) is the most effective treatment for obstructive sleep apnea syndrome. Polysomnography studies will help in the quantification of the severity of OSA in the form of a respiratory disturbance index (RDI). CPAP is generally prescribed for patients with an RDI > 20 or those with lower RDIs with additional symptoms such as daytime somnolence. Overall n-CPAP such as daytime somnolence. Overall n-CPAP has been found to ameliorate daytime sleepiness, depressive symptoms, and elevated diastolic blood pressure.
145
Q

MED 80. Which test is used to diagnose central and obstructive sleep apnea?
A) Polysomnography
B) Spirometry with pulmonary function testing
C) Diffusion capacity testing
D) Pulse oximetry

A
  1. A) The “gold standard” for diagnosing sleep apnea is an overnight polysomnography (PSG or “sleep study”). Such studies are typically performed in the outpatient setting under the observation of a qualified technician. Indications for such a study include excessive daytime sleepiness, titration of continuous positive airway pressure (CPAP) therapy, unexplained pulmonary hypertension (HTN), or poorly controlled HTN, among other PSG monitors sleep stages by monitoring an electroencephalograph, an electrooculography, respiratory flow/effort, oxyhemoglobin saturation, an electrocardiogram, and body position. PSG results will help to categorize central from obstructive sleep apnea.
146
Q
MED 85.  Which lung volume increases in cervical spinal cord injury (SCI)?
       A)  Residual volume
       B)  Total lung capacity
       C)  Vital capacity
       D)  Tidal volume
A
  1. A) Persons with SCI typically have a restrictive form of pulmonary disease where all lung volumes are reduced except for the residual volume (RV), which increases.
147
Q

MED 86. In the three-zone model of the lung, which zone has the highest pulmonary arterial pressure (PAP) when upright?
A) Zone 1
B) Zone 2
C) Zone 3
D) None, they all have equivalent hydrostatic pressure

A
  1. C) In the three-zone model of pulmonary blood flow, the interrelationship among alveolar pressure (PA), pulmonary arterial pressure (Pa), and pulmonary venous pressure (PV) is described. In zone 1, PA exceeds Pa, and therefore ventilation occurs in excess of perfusion. In zone 2, the following relationship occurs where Pa > PA > PV, so ventilation and perfusion are fairly equal. Finally, in zone 3, Pa > PV > PA, so flow is dependent on the arterial and venous pressure difference and has the highest rate of perfusion relative to the other two zones. Of course, perfusion of the lung is dependent on posture, and the above is assuming an upright posture.
148
Q
MED 89.  What intervention minimizes the reduction in vital capacity of tetraplegic patients when they are sitting?
       A)  Use of an abdominal binder
       B)  Supplemental oxygen
       C)  Glossopharyngeal breathing
       D)  Compressive leg stockings
A
  1. A) In C5 tetraplegic patients, some innervation to the diaphragm remains intact. However, the abdominal contents may tend to sag secondary to weakened abdominal muscles. When tetraplegic patients go from a supine to sitting posture, the diaphragmatic excursion decreases as it becomes pulled down by the sagging abdominal contents and the vital capacity reduces. It has been found that using an abdominal binder while sitting may reduce this drop in vital capacity.
149
Q

MED 91. What is a contraindication for chest percussion therapy?
A) Anticoagulation therapy
B) Increased intracranial pressure
C) Flail chest
D) Severe osteoporosis

A
  1. B) Chest percussion involves the use of a cupped hand or mechanical percussor and providing rhythmic strikes to the chest wall and rib cage to loosen mucous. Contraindications for such therapy include increased intracranial pressure, increased intraocular pressure, cardiovascular collapse or instability, aortic aneurysm, and gross tumor. Precautionary measures should be taken for conditions where patients are on anticoagulation therapy, have thrombocytopenia, rib fractures, or severe osteoporosis; however, these are not contraindications.
150
Q
MED 92.  Which tracheostomy tube is appropriate for patients able to speak who only require intermittent ventilator assistance?
       A)  Cuffed tracheostomy tube
       B)  Nonfenestrated tube
       C)  Passy-Muir valve
       D)  Fenestrated tube
A

D) Tracheostomy tubes have various features depending on what is needed for the patient. Cuffed tubes allow for good air seals and protection from aspiration, but do not allow the patient to speak. Uncuffed tubes provide a looser fit and poorer air seal, but allow for the patient to verbally communicate when needed. Fenestrated tubes are appropriate for patients able to speak and who intermittently require ventilator support. Nonfenestrated tubes are for patients requiring continuous mechanical ventilation or who cannot protect the airway from aspiration. Passy-Muir valve is a brand of talking tube that has a one-way valve to facilitate speech with a tracheostomy.

151
Q

MED 95. During normal inspiration, what is the action of the vocal cords?
A) The vocal cords should open
B) The vocal cords should close
C) The vocal cords should close then open
D) The vocal cords remain inactive during inspiration

A
  1. A) During normal respiration, the vocal cords partially abduct (open) during inspiration and partially adduct (close) during end-exhalation. In vocal cord dysfunction (paradoxical vocal cord motion), the vocal cords adduct during inhalation, resulting in airflow obstruction at the level of the larynx. This diagnosis can be confirmed by visualization with a laryngoscope.
152
Q

MED 96. What technique uses gravity to assist in the ultimate clearance of secretions from specific lung areas?
A) Abdominal binder uses
B) Manual suction through tracheostomy
C) Postural drainage
D) Abdominal thrust

A

C) Postural drainage uses gravity-assisted positioning to clear the airways. Common positions include Trendelenburg with the patient side-lying, supine, or prone depending on which lobe is being drained. Such techniques should be avoided in patients with significant pulmonary edema, congestive heart failure, acute dyspnea, or aspiration problems.

153
Q

MED 99. What describes “Ondine’s curse”?
A) Acquired central hypoventilation syndrome
B) Congenital central hypoventilation syndrome
C) High cervical spinal cord injury resulting in severe diaphragmatic impairment
D) Polio syndrome affecting the upper trunk more than the lower trunk and legs

A

. B) The literary misnomer known as “Ondine’s curse” refers to a rare hereditary disorder characterized by congenital central hypoventilation. Recent research suggests that this disorder is more consistent with a generalized dysfunction of the regulation of the body’s autonomic nervous system. Patients usually present in infancy with respiratory distress soon after birth requiring mechanical assistance. They may also develop more normal patterns of breathing during wakefulness with episodes of apnea during sleep.

154
Q

MED 103. Which ventilator setting coordinates delivery of the ventilator-driven breath with the respiratory cycle of the patient?
A) Assist-control ventilation (ACV)
B) Intermittent mandatory ventilation (IMV)
C) Pressure support ventilation (PSV)
D) Synchronized intermittent mandatory ventilation (SIMV)

A
  1. D) SIMV allows the ventilator to become sensitized to the patient’s respiratory pattern and deliver ventilator-assisted breaths in synchrony with the patient. ACV is usually the initial mode of ventilation for patients intubated for respiratory failure. The ventilator is set at a selected backup rate and is able to deliver breaths for every patient-initiated effort as well as when the patient’s respiratory rate falls below the preset backup rate. PSV provides augmentation of patient’s spontaneous respiratory effort and can be used during weaning trials.
155
Q

MED 104. What is PEEP?
A) Fraction of inspired oxygen
B) The patient’s respiratory rate at a given tidal volume
C) Positive end-expiratory pressure
D) Adjunct to conventional modes of mechanical ventilation to decrease work of breathing

A
  1. C) Positive airway pressure at the end of expiration is pressure above atmospheric pressure mechanically exerted on the airways to keep them patent. The overall effect of PEEP if applied during mechanical ventilation is to increase lung compliance and oxygenation while decreasing shunt fraction and the work of breathing.
156
Q
MED 105. What is the leading cause of mortality in chronic tetraplegic spinal cord injury (SCI) patients?
       A)  Decubitus ulcers
       B)  Urinary complications
       C)  Deep vein thrombosis (DVT)
       D)  Pneumonia
A
  1. D) Pneumonia is the leading cause of mortality among SCI patients, particularly those who are tetraplegics. Heart disease is the second most common cause, whereas septicemia (from pressure ulcers, urinary tract infections, or respiratory infections) is third.
157
Q

MED 106. Which test is useful in evaluating the phrenic nerve?
A) Electrodiagnostic studies (EMG/NCS)
B) Computed tomography (CT)
C) Magnetic resonance imaging (MRI)
D) Ultrasound

A
  1. A) Electrodiagnostic studies of the phrenic nerve are necessary to determine whether a patient is a candidate for phrenic nerve pacing. Phrenic pacing is an increasingly used method to provide ventilation to high-level spinal cord injury (SCI) patients (C1-C2) besides chronic ventilator use. Ideal candidates for pacing have an intact diaphragm and phrenic nerve, but impaired respiratory function because of a higher lesion (as in central hypoventilation syndrome, brainstem injury, or high [C1-C2] cervical lesions).
158
Q
MED 107. Where are the central respiratory control centers located?
       A)  Thalamus
       B)  Hippocampus
       C)  Cortex
       D)  Medulla
A

D) Central respiratory centers are located in the medulla (specifically in the dorsal respiratory group) and initiate the rhythm of breathing. Other respiratory centers are located in other parts of the medulla as well as the pons.

159
Q
MED 108. Central chemoreceptors of respiratory regulation are sensitive to which of the following levels?
       A)  Hydrogen ions
       B)  PO2
       C)  PCO2
       D)  Both A and C
A

D) Central chemoreceptors in the medulla are sensitive to both changes in pH and rising CO2 levels within the cerebrospinal fluid of the fourth ventricle. There are peripheral chemoreceptors as well, which are sensitive to pH, pCO2, and pO2 levels. Such receptors are located in the aortic arch as well as the carotid bodies.

160
Q

MED 111. What is glossopharyngeal breathing (GPB)?
A) Breathing through a tracheostomy
B) Functional electrical stimulation technique used on the pharyngeal muscles
C) Breathing technique used in the event of ventilator failure where a patient takes in several boluses of air
D) Airway secretion clearance technique

A
  1. C) Patients who have severely limited vital capacity and inspiratory muscle weakness can be trained in the technique of GPB. This strategy involves the patient being able to take in several successive “gulps” of air and closing the glottis after each one. With proper coordination, patients can manage to take in several hundred milliliters of air and thereby maintain alveolar ventilation while off of the ventilator. This technique cannot be used by patients with a tracheostomy or even a capped tracheostomy because of air leakage around the stoma.
161
Q

MED 112. What is an intermittent abdominal pressure ventilator (IAPV)?
A) A method of introducing air into the abdomen to manually raise the diaphragm
B) A treatment for obstructive sleep apnea
C) A cough assist device that exerts abdominal thrusts
D) A daytime inspiratory muscle aid worn underneath the clothing

A

D) IAPV is a body ventilator that consists of an elastic air sac, which is worn underneath the clothing by patients who have respiratory muscle weakness. When the elastic bladder inflates with air by an external positive pressure ventilator, it causes the diaphragm to move upward. Once it deflates, the abdominal contents sag and pull the diaphragm down, allowing inspiration to occur passively. This technique can augment tidal volume by at least 300 mL or even higher if the patient has any degree of inspiratory capacity.

162
Q

MED 114. What is VO2max?
A) Maximal oxygen uptake and use by the body during exercise
B) Fraction of inspired oxygen necessary to maintain SpO2 > 90%
C) Maximum volume of oxygen necessary to carry out a designated activity
D) Represents the arteriovenous oxygen difference

A

A) VO2max (maximum aerobic capacity or maximal oxygen consumption) represents the maximum ability of an individual to take in, consume, and utilize oxygen. This value defines an individual’s level of physical fitness. This value is defined by the Fick equation: VO2 = HR × SV × a-VO2diff. VO2max is considered the single best measure of cardiopulmonary fitness.

163
Q

MED 115. What is the peak cough flow (PCF)?
A) The maximum volume of air a patient can hold with a closed glottis
B) A breathing technique that can be taught to patients with neuromuscular weakness
C) A method to check for intact gag reflex prior to extubation
D) The velocity of air expelled from the airways during a cough maneuver

A

D) PCF can be measured using a peak flow meter, and it is measured in L/min or L/ sec (L/min divided by 60 seconds). Measuring the PCF is an acceptable way to measure expiratory muscle force, particularly in patients who have muscle weakness (as in Duchenne muscular dystrophy or amyotrophic lateral sclerosis). This value correlates with what is called the maximal expiratory pressure (MEP), another indicator of expiratory muscle force, which is harder to measure in patients with facial muscle weakness. Measuring and tracking the PCF and MEP in patients with neuromuscular weakness is important in assessing such patients’ ability to produce efficient and productive coughs for the clearance of airway irritants and/or secretions. Weakness or inability to produce this physiologic mechanism can result in respiratory infection and ultimate failure.

164
Q

MED 130. Most neoplastic spinal cord compression is:
A) Extramedullary
B) Intramedullary
C) The incidence of intramedullary and extramedullary involvement is about equal

A
  1. A) Extramedullary (outside cord) compression accounts for more than 90% of spinal cord compression resulting from cancer. It includes epidural, intradural, and leptomeningeal disease. Intramedullary (inside the cord) spinal cord compression is seen in less than 5% of spinal cord tumors and includes gliomas, ependymomas, and astrocytomas.
165
Q
MED The earliest presentation of neoplastic spinal cord compromise is:
       A)  Pain
       B)  Weakness
       C)  Sensory loss
       D)  Bowel and bladder dysfunction
A

A) Pain is usually the classic presenting symptom, preceding the others. An acute onset of back pain in a patient with cancer is metastasis until proven otherwise. Pain is usually progressive, not relieved with rest and worse at night. Weakness and bowel/bladder dysfunction in a cancer patient is suspicious for spinal cord compression.

166
Q
MED 139. The most common site of metastasis in the spine is:
       A)  Vertebral body
       B)  Vertebral foramen
       C)  Pedicle
       D)  Lamina
A
  1. A) The most common site of vertebral metastases is the vertebral body, most commonly in the thoracic spine.
167
Q

MED 140. What is the best modality for detecting spinal metastases?
A) Magnetic resonance imaging (MRI)
B) Computed tomography (CT)
C) X-ray
D) Bone scan

A

. A) MRI remains the best diagnostic tool for detecting spinal metastases. It has a sensitivity of 83% to 93% with a specificity of 90% to 97%. The use of diffusion-weighted and contrast enhancement allows for differentiation between benign and malignant tissue.

168
Q
MED 141. Complications of bone metastases include:
       A)  Hypercalcemia
       B)  Spinal cord compression
       C)  Pathological fracture
       D)  All of the above
A
  1. D) Pathologic fractures can be a complication of or the initial presentation of bone metastases. They usually occur as a result of the progression of bony disease. Hypercalcemia in patients with cancer is primarily due to increased bone resorption and the release of calcium from bone. Epidural metastasis is the most ominous complication of bone metastasis to the vertebral spine and is a medical emergency. The tumor enters the epidural space by contiguous spread from adjacent vertebral metastasis. Failure to properly diagnose this condition can lead to spinal cord injury and permanent neurological damage. Signs of spinal cord compression include motor, sensory, and autonomic (bladder and bowel) dysfunction.
169
Q
MED 142. Bony metastases are common in cancer of the:
       A)  Breast
       B)  Prostate
       C)  Lung
       D)  All of the above
A
  1. D) Breast, prostate, and lung are the most common cancers to metastasize to bone.
170
Q
MED 148. Common side effects of bisphosphonates include:
       A)  Hypocalcemia
       B)  Renal toxicity
       C)  Atypical femur fractures
       D)  Gastrointestinal irritation
A

D) Bisphosphonates can cause inflammation of the esophagus and lead to erosions of the surface of the esophagus. Renal toxicity rarely occurs and is usually seen in patients who already have significant renal impairment. Atypical femur fractures are very rare fractures in patients taking bisphosphonate therapy for prolonged periods (> 5 years). Although studies vary, some show that bisphosphonate use was associated with a 47% lower risk for classic fracture in patients with osteoporosis.

171
Q
MED 149. Bisphosphonates prevent fractures by:
       A)  Inhibiting osteoclasts
       B)  Promoting osteoblasts
       C)  Remodeling bone
       D)  All of the above
A
  1. A) Bisphosphonates inhibit osteoclastic bone resorption.
172
Q
MED 162. The most common site of spinal metastases is:
       A)  Thoracic spine
       B)  Cervical spine
       C)  Lumbar spine
       D)  All are equal
A
  1. A) The most common site of metastases is the vertebral body of the thoracic spine (70%).
173
Q

MED 165. The spine is considered unstable when there is tumor involvement of the:
A) Middle column
B) Two or more columns are involved
C) Either A or B
D) The anterior column

A

C) When assessing spine stability, the spine can be divided into three columns. The anterior column consists of anterior longitudinal ligament, anterior annulus, and anterior 2/3 of vertebral body. The middle column consists of the posterior 1/3 of the vertebral body, posterior annulus, and posterior longitudinal ligament. The posterior column consists of the pedicles, facets, lamina, spinous process, and posterior ligaments. The spine is considered stable if only one column is affected, unless it is the middle column. It is considered unstable if two or more columns are involved. It is also considered unstable if there is a 20 degree or greater angulation. The spine stability system was developed for the traumatic spine. However, the issues are more complex in cancer. Metastatic bone disease is a dynamic disease, with both mechanical and chemical forces acting on the bone.

174
Q
MED 166. An unstable metastatic bony lesion is best treated by:
       A)  Surgical fixation
       B)  Radiation therapy
       C)  Physical therapy
       D)  None of the above
A
  1. A) An unstable metastatic lesion is best treated with fixation. Much has been done to try to develop a way to tell which lesions are unstable. Lytic lesions that measure more than 2/3 of the cortex and those that present with functional pain are at greater chance for pathologic fracture.
175
Q
MED 186. Which is the most common tumor site associated with malignant spinal cord compression?
       A)  Epidural
       B)  Spinal cord
       C)  Intradural
       D)  Vessels
A
  1. A) Epidural tumors most commonly produce spinal cord compression.
176
Q
MED 187. What is the pathophysiology of radiation-induced transient myelopathy?
       A)  Inflammation
       B)  Transient demyelination
       C)  Local edema
       D)  Fibrosis
A
  1. B) The main mechanism of radiation-induced myelopathy is transient demyelination.
177
Q
MED 188. Which of the following symptoms indicates delayed radiation myelopathy?
       A)  Lower extremity paresthesias
       B)  Deep vein thrombosis
       C)  Gangrene
       D)  All of the above
A
  1. A) The symptoms of delayed radiation myelopathy include lower extremity paresthesias, weakness, and pain.
178
Q

MED 198. Which of the following is a manifestation of bone metastases in patients with multiple myeloma?
A) Fracture
B) Lytic lesions with sclerotic borders
C) Bone pain
D) All of the above

A
  1. D) Patients with multiple myeloma and bone metastasis may suffer from bone fracture, osteoporosis, bone pain, hypercalcemia, and/or lytic lesions with sclerotic borders.
179
Q
MED 197. Which cancer is associated with blastic bone metastases?
       A)  Lung
       B)  Melanoma
       C)  Prostate
       D)  Colon
A
  1. C) Prostate cancer is associated with blastic bone metastases.
180
Q

MED 196. Which diagnostic test is the most sensitive test for bone metastases?
A) X-ray
B) Bone scan
C) Computed tomography (CT)
D) Magnetic resonance imaging (MRI)

A
  1. B) The most sensitive diagnostic test for the detection of bone metastases is bone scan.
181
Q
MED 151. Multiple myeloma is caused by the proliferation of:
       A)  Plasma cells
       B)  Red blood cells
       C)  White blood cells
       D)  None of the above
A
  1. A) In multiple myeloma, collections of abnormal plasma cells accumulate in the bone marrow, where they produce a paraprotein, which can cause kidney disease, bone lesions (lytic), and hypercalcemia.
182
Q
MED 153. Bone scans can assess:
       A)  Blastic activity
       B)  Lytic activity
       C)  Bone destruction
       D)  Impending fracture site
A
  1. A) Tracer accumulates in new bone that is formed in response to a new lesion and can occur at any site with bone turnover. It is extremely nonspecific; only 50% of foci represent cancer. Positive scans require further radiologic confirmation.
183
Q
MED 154. Bone lesions that are not assessed by bone scan include:
       A)  Prostate cancer
       B)  Multiple myeloma
       C)  Breast cancer
       D)  All of the above
A
  1. B) Bone scan will be negative in lytic lesions, as seen in multiple myeloma. Bone scans should be ordered along with x-ray.
184
Q
MED 155. Which cancer presents with lytic lesions?
       A)  Multiple myeloma
       B)  Non–small lung cell cancer
       C)  Renal cancer
       D)  All of the above
A
  1. D) Lytic lesions predominate in the above cancers as well as cancers of melanoma, thyroid, and non-Hodgkin’s lymphoma.
185
Q
PO 122. Which anatomic structure should be used as a reference point in order to determine the proper backrest height for a standard wheelchair?
       A)  Iliac crest
       B)  Posterior 12th rib
       C)  Scapula
       D)  T1 vertebrae
A

. C) Generally, the backrest should be low enough to provide support and still allow the upper limbs access to the pushrims. The scapula, particularly the inferior angle of the scapula, is used to determine the proper backrest height on a standard wheelchair. The scapula connects the humerus with the clavicle. There are 18 muscles that attach to the scapula, which are indirectly related to the movement of the upper limbs. These include pectoralis minor, long head of triceps brachii, short head of biceps brachii, long head of biceps brachii, coracobrachialis, serratus anterior, subscapularis, rhomboid major, rhomboid minor, levator scapulae, trapezius, deltoid, supraspinatus, infraspinatus, omohyoid, latissimus dorsi, teres minor, and teres major. It is essential, if no cervical pathology is involved, that the backrest be measured to below the inferior angle of the scapula in order to avoid any somatic restriction of scapula.

186
Q
PO 121. When measuring a patient for a wheelchair, how many inches should the wheel chair width be in relation to the widest part of the buttocks?
       A)  0 inches
       B)  1 inch greater
       C)  2 inches greater
       D)  3 inches greater
A

B) The wheelchair seat width should be approximately 1 inch wider than the width of the widest part of the buttocks. If the distance is too narrow, as in answer choice A, pressure ulcers may develop from the pelvic bony prominences being in close contact to the sides of the chair. Answer choices C and D are incorrect because if the distance is too wide, the individual will have to markedly abduct the shoulders making it more difficult to push the chair.

187
Q

PO 116. This cervical orthosis is useful for bed-ridden patients with a C1 fracture since it does not have posterior uprights:
A) Minerva brace
B) Miami collar
C) Sterno-occipital-mandibular immobilizer (SOMI)
D) Halovest

A

C) The SOMI has three posters with an anterior chest plate that extend over the shoulder. One of the posters extends from the chest plate to the chin and the other two extend to the occiput. It is useful in bed-ridden patients since it has no posterior posters. However, this also limits its effectiveness for restricting extension. It is very effective in limiting flexion, lateral bending, and rotation.

188
Q
PO 115. This lumbosacral brace has two posterior uprights that prevent extension and an anterior apron with two anterior straps; one at the thorax and one at the pelvis that limit flexion:
       A)  Rainey jacket
       B)  Clam shell brace
       C)  Milwaukee brace
       D)  Chairback brace
A

D) The Chairback brace is used on patients with low back pain from disc herniations by preventing extension of the lumbar spine. It also helps decrease lumbar lordosis by increasing intra-abdominal pressure by tightening the anterior straps. It is also used as an immobilizer after a lumbar laminectomy.

189
Q
PO 114. This brace is used in patients with spondylolisthesis. It restricts extension, but allows flexion, since the front is made up of an elastic band. It also has side uprights that prevent lateral bending:
       A)  Williams brace
       B)  Chairback brace
       C)  Rainey jacket
       D)  Taylor brace
A
  1. A) The Williams brace is a short lumbosacral brace that helps with spondylolisthesis as well as spondylolysis by restricting extension.
190
Q

PO 98. Which is a potential complications of spinal orthoses?
A) Skin ulcers
B) Weakening of axial muscles
C) Physical and psychological dependence
D) All the above

A
  1. D) Other complications include soft tissue contractures, increased movement at the ends where the spine is not immobilized, and osteopenia.
191
Q
PO 99.  Which spinal segment has the greatest range of motion?
       A)  Cervical
       B)  Thoracic
       C)  Lumbar
       D)  Sacral
A
  1. A) The cervical spine has the greatest range of motion including flexion, extension, side bending, and rotation.
192
Q
PO 100. Where does most of cervical rotation occur?
       A)  C1-C2
       B)  C3-C4
       C)  C4-C5
       D)  C6-C7
A
  1. A) Forty-five percentage of cervical rotation occurs at the atlanto-axial joint, where the atlas rotates around the odontoid process of the axis.
193
Q
PO 101. Where does the greatest amount of cervical flexion occur?
       A)  C2-C3
       B)  C3-C4
       C)  C5-C6
       D)  C6-C7
A
  1. C) The greatest amount of cervical flexion occurs at C5-C6. C4-C5 accounts for the second most amount of cervical flexion
194
Q
PO 102. Which lumbar segment is the most mobile?
       A)  L2-L3
       B)  L4-L5
       C)  L3-L4
       D)  L1-L2
A
  1. B) About 80% to 90% of lumbar motion occurs at L4-L5 and L5-S1. This also makes these segments the most susceptible to degenerative changes.
195
Q

PO 103. Which of the following cervical orthosis does not limit rotation?
A) Halovest
B) Minerva brace
C) Aspen collar
D) Sterno-occipital-mandibular immobilizer (SOMI)

A
  1. C) The Aspen collar limits cervical flexion and lateral bending. However, it does not limit rotation. The Halovest, Minerva, and SOMI also limit flexion and side-bending as well.
196
Q
PO 104. A patient with a C2 fracture is placed on a Halovest. How long does he have to wear the vest?
       A)  6 weeks
       B)  9 weeks
       C)  12 weeks
       D)  18 weeks
A
  1. C) The Halo is usually worn for about 12 weeks to allow for adequate bone healing. Complications of this orthosis include pin loosening, infection, ring migration, pressure sores and pin scars.
197
Q
PO 105. A patient has a stable C4 fracture. Which cervical orthosis would be appropriate?
       A)  Soft collar
       B)  Philadelphia collar
       C)  Halovest
       D)  Minerva
A
  1. B) The Philadelphia collar limits flexion, extension, and side-bending. It is used for cervical ligament ruptures and relatively stable cervical fractures. It is also used after cervical surgery when strict neck immobilization is not necessary.
198
Q
PO 106. Which of these back braces is primarily used for stable vertebral anterior compression fractures?
       A)  Williams brace
       B)  Chairback brace
       C)  Taylor brace
       D)  Knight brace
A
  1. C) The Taylor brace is a thoracolumbosacral orthosis (TLSO) that limits flexion and extension. It is used to prevent worsening of kyphosis from a stable anterior compression fracture of the vertebra.
199
Q
PO 107. Which truncal orthosis uses a three-point pressure system to allow for extension but limit flexion?
       A)  Jewett brace
       B)  Williams brace
     C)  Taylor brace
       D)  Clam shell brac
A
  1. A) The Jewett brace applies pressure to the manubrium, midback, and pubic symphysis, which allows for extension, but limits flexion.
200
Q

PO 108. Which orthotics is used for low back pain during pregnancy?
A) Lumbosacral orthosis (LSO) with a rigid frame
B) Corset
C) Wide belt
D) Rainey orthosis

A
  1. C) Also known as a sacroiliac orthosis, these bands fit between the iliac crests and the greater trochanters of the femur. It avoids application of pressure on the pelvis while providing support to the sacroiliac (SI) joint as well as the ligaments around it that are strained during pregnancy.
201
Q
PO 109. What is the highest thoracic spine level for which a clam shell brace is effective in immobilizing the lower trunk?
       A)  T1
       B)  T5
       C)  T8
       D)  T11
A
  1. C) The clam shell orthosis is a custom molded body thoracolumbosacral (TLS) body jacket that provides the greatest lower truncal immobilization. It is usually used after thoracolumbar fixations. It is effective only as high as T8.
202
Q

PO 110. Which of these thoracolumbosacral orthoses (TLSO) cannot be used on an unstable thoracolumbar spine fracture?
A) Jewett brace
B) Cruciform anterior spinal hyperextension (CASH) brace
C) Molded plastic TLSO
D) Taylor brace

A
  1. D) The Taylor brace is only used for stable compression fractures.
203
Q
PO 111. Which of these braces is used for low thoracic scoliosis?
       A)  Jewett brace
       B)  Milwaukee brace
       C)  Knight brace
       D)  Williams brace
A
  1. B) The Milwaukee brace consists of three uprights (2 posterior and 1 anterior) that attach onto a ring around the neck and pelvic girdle. Straps with pressure pads are mounted onto the brace to help improve spinal alignment.
204
Q

PO 112. Which of these orthotics is used to decrease kyphosis from a stable vertebral anterior compression fracture by applying an anterior crossbar to the torso?
A) Taylor brace
B) Jewett brace
C) Cruciform anterior spinal hyperextension (CASH) brace
D) Boston brace

A
  1. C) The cruciform anterior spinal hyperextension orthosis uses an anterior cross with padding at the four ends of the cross to prevent further progression of kyphosis from compression fractures by limiting flexion while allowing extension. The brace is adjusted posteriorly with Velcro straps.
205
Q

PO 113. This cervical orthosis consists of one anterior poster supporting the chin and one posterior poster supporting the occiput that attaches to a thoracic jacket:
A) Rainey orthosis
B) Sterno-occipital-mandibular immobilizer (SOMI)
C) Minerva brace
D) Aspen collar

A
  1. C) The Minerva brace is an alternative to the Halovest and is used for cervical spine fractures below C2. It restricts flexion/extension, side bending, and rotation similar to the Halovest. However, it is not used for fractures at C1-C2.
206
Q

PO 89. What would you not tell a paraplegic patient who is inquiring about using a knee ankle foot orthosis (KAFO)?
A) Using a KAFO requires a lot of energy
B) Your KAFO can replace your wheelchair as your primary means of locomotion in time
C) You should use forearm crutches while walking with a KAFO
D) Ambulation with a KAFO uses a swing through gait

A
  1. B) KAFOs are meant to be an adjunct to locomotion for paraplegics. They are not meant to replace the use of a wheelchair.
207
Q
PO 91.  Where is the ground reaction force (GRF) placed with an offset knee joint on a knee ankle foot orthosis (KAFO)?
       A)  Medial
       B)  Lateral
       C)  Anterior
       D)  Posterior
A
  1. C) The hinge is placed posterior to the knee’s axis of rotation so that the GRF is anterior. This helps with extension of the knee for stability during early stance phase.
208
Q
PO 78.  A patient has a weakened peroneus longus muscles. What kind of foot orthotic should be placed in his shoe to prevent his ankle from rolling?
       A)  Medial wedge
       B)  Lateral wedge
       C)  Heel cushion
       D)  Rocker bar
A

B) The peroneus longus muscle helps with pronation of the foot. Thus, if this muscle was weak, a patient will be prone to supination of the foot. Placing a lateral wedge would help this. A medial wedge is used for patients with weak tibialis posterior who have the opposite problem and have difficulty supinating

209
Q
PO 80.  Which of these devices assists with plantar flexion?
       A)  Anterior stop
       B)  Posterior stop
       C)  Anterior spring
       D)  Posterior spring
A
  1. C) Stops are devices that inhibit certain movements, whereas springs assist certain movements. Anterior stops inhibit dorsiflexion, whereas posterior stops inhibit plantar flexion. Anterior springs aid with plantar flexion, whereas posterior springs help with dorsiflexion.
210
Q

PO 81. Which one of these is an indication for using an ankle foot orthosis (AFO) to improve a patient’s gait?
A) Amputation of the great toe
B) Weak push off at late stance phase
C) Anteroposterior instability at the ankle
D) Foot fracture

A
  1. B) Other indications for using an AFO include foot drop during swing phase, mediolateral instability at the ankle, and foot slap during heel strike due to weak ankle dorsiflexors.
211
Q
PO 82.  Which type of ankle foot orthosis (AFO) is used when clonus is present at the ankle?
       A)  Solid AFO
       B)  Posterior assist
       C)  Anterior assist
       D)  All of the above
A
  1. A) Springs may worsen ankle clonus. Therefore, using a solid AFO helps lock the ankle and control clonus.
212
Q
PO 83.  Which of the following is an ankle foot orthosis (AFO) with the footplate set in slight plantar flexion to help with stability at the knee during full extension for patients with weak quadriceps?
       A)  Posterior assist AFO
       B)  Posterior stop AFO
       C)  Anterior stop AFO
       D)  Floor reaction AFO
A
  1. D) This orthosis has a band of material on the top of the orthosis that helps transfer the extension moment from plantar flexing to the patellar tendon. This helps stabilize the knee during heel strike.
213
Q

PO 84. Which ankle foot orthosis (AFO) can be used for someone with a foot drop and weak quadriceps?
A) AFO with posterior foot stop
B) Solid AFO
C) AFO with posterior stop with a Klenzak joint
D) Solid AFO with anterior and posterior stops

A

C) Answer option A would be good for a foot drop, but would not address the weak quadriceps. Answer option B is useful for pain with ankle motion. Answer option D is useful for weak plantar flexors.

214
Q

A patient wearing a new ankle foot orthotic (AFO) starts complaining of numbness on his foot. He says that it feels a little tight on the upper part of the orthosis. You notice that the top of the AFO is at the level of the fibular head. What can you suggest to this patient?
A) Widen the upper brim of the orthosis
B) Add some padding to the upper brim
C) Find another orthotist
D) Adjust the AFO by taking away 1 inch from the upper brim

A
  1. D) This patient is having his peroneal nerve compressed at the fibular head. The AFO should have a 1- to 2-inch clearance from the upper brim to the fibular head.
215
Q
PO 87.  What can be placed on an ankle foot orthotic (AFO) for someone with weak mediolateral ankle instability?
       A)  Double metal upright
       B)  T strap
       C)  B strap
       D)  A and B
A
  1. D) T straps are placed on the ankle with double metal upright AFOs to enhance mediolateral stability at the ankle.
216
Q
PO 88.  What type of T strap is used for someone with a valgus ankle deformity?
       A)  Medial T strap
       B)  Lateral T strap
       C)  Superior T strap
       D)  Posterior T strap
A
  1. A) The medial T strap attaches onto the shoe over the medial side and attaches onto the lateral upright of the double metal upright. Answer option B (lateral T strap) would be used for a varus deformity/laxity.
217
Q

PO 93. Which one of these does not promote knee extension?
A) Cushioned heel
B) Ankle plantar flexion
C) Anterior ground reactive force
D) Posterior stop

A
  1. D) Posterior stops limit plantar flexion and would actually promote knee flexion rather than extension.
218
Q
PO 94.  You notice that your patient has a valgus deformity at the ankle when he walks. What adjustments to his shoe can you suggest so that his gait is more stable?
       A)  Cushioned heel
       B)  Medial heel flare
       C)  Lateral heel flare
       D)  Heel lift
A
  1. B) Medial heel flares resist valgus motion at the ankle, whereas lateral heel flares resist varus motion.
219
Q

PO 95. A patient has weak plantar flexors. What adjustments to his shoes can you make to assist him?
A) A rocker bar at the sole of the shoe
B) Thomas orthopedic heel
C) Medial sole and heel wedging
D) Reverse Thomas heel

A
  1. A) The rocker bottom relieves pressure from the metatarsal heads and assists in rollover when plantar flexors are weak.
220
Q

PO 96. Which of the following does not promote knee flexion?
A) Cushioned heel
B) Posterior stop
C) Posterior ground reactive force
D) Ankle dorsiflexion

A
  1. A) A cushioned heel helps promote plantar flexion. This would promote knee extension rather than knee flexion.
221
Q
PO 97.  What adjustments to the shoe would be good for a patient with pes planus?
       A)  Lateral sole flare
       B)  Thomas heel
       C)  Reverse Thomas heel
       D)  Rocker bottom sole
A
  1. B) Thomas heel extends anteriorly under the navicular bone and provides support to the medial longitudinal arch for patients with pes planus.
222
Q
Pain  4.  Which type of nerve fibers transmit the first sensation of pain?
       A)  A delta fibers
       B)  C fibers
       C)  A beta fibers
       D)  B fibers
A
  1. A) A delta fibers (group III fibers) are 2 to 5 mm in diameter, are myelinated, have a fast conduction speed (5–40 meters/sec), and are the first fibers to transmit the sensation of pain.
223
Q

PA 2. Allodynia is:
A) Pain resulting from a stimulus that does not normally produce pain
B) An increased painful sensation in response to additional noxious stimuli
C) A decreased sensitivity to painful stimuli
D) The absence of the sense of pain while remaining conscious

A
  1. A) Allodynia is pain resulting from a stimulus that does not normally produce pain. Hyperalgesia is an increased painful sensation in response to additional noxious stimuli. Analgesia is defined as the absence of the sense of pain without losing consciousness and other sensations. Hypoalgesia or hypalgesia is a decreased sensitivity to painful stimuli.
224
Q

IR 55. Clinical symptoms of cauda equina syndrome include all of the following except:
A) Radicular pain and numbness involving both legs
B) Loss of perineal sensation/saddle anesthesia
C) Unaffected gait pattern
D) Urinary frequency, urinary retention or incontinence

A
  1. C) Cauda equina typically involves radicular pain and numbness involving both legs, although symptoms may be more severe in one extremity. Leg weakness may present with foot drop, stumbling gait, or difficulty getting up from a chair. Pain in the lower extremities may diminish as the paralysis progresses. There may be difficulty voiding or loss of urinary or anal sphincter control. Saddle anesthesia/loss of perineal sensation is typical.
225
Q
IR 50.  Heel walking tests motor strength of the ankle dorsiflexors and involves the following nerve root(s):
       A)  S1
       B)  L4–5
       C)  L3–4
       D)  L1–2
A
  1. B) Heel walking and toe walking are utilized to evaluate muscle strength and spinal innervation. Heel walking is a quick screen for L4–5 aggravation, and toe walking is a screen for S1 innervation.
226
Q
PA   8.  Which nerve innervates the L4/L5 facet joint?
       A)  L2 and L3 medial branches
       B)  L3 and L4 medial branches
       C)  L4 and L5 medial branches
       D)  L4 dorsal ramus
A

B) Each lumbar and thoracic facet joint (except L5/S1 facet joint) is innervated by the medial branches of dorsal rami exiting at the same level and one level above. The L4/L5 facet-joint is innervated by the L3 and L4 medial branches.

227
Q
PA 14.  The cell bodies of first-order, or primary, thoracic visceral pain fibers are found in:
       A)  Dorsal root ganglion
       B)  Trigeminal ganglion
       C)  Mesenteric ganglion
       D)  Inferior cervical ganglion
A
  1. A) The cell bodies of first-order, or primary, pain fibers are located in either the dorsal root ganglia or the trigeminal ganglia. The trigeminal ganglia are specialized nerves for the face, whereas the dorsal root ganglia provide sensory innervation for the rest of the body.
228
Q
PA 15.  The initial gate control theory by Melzack and Wall proposed that stimulation of \_\_\_\_\_ fibers modulates the dorsal horn “gate” and therefore reduces the nociceptive input from the periphery.
       A)  A beta
       B)  B
       C)  C
       D)  A delta
A
  1. A) The initial gate control theory by Melzack and Wall published in Science in 1965 indicated that stimulation of large diameter A beta fibers modulated the dorsal horn “gate” and therefore reduced the nociceptive input from periphery.
229
Q

PA 16. Which of the following is not an application based on the gate control theory?
A) Spinal cord stimulation
B) Massage
C) Transcutaneous electrical nerve stimulation (TENS)
D) Medial branch block

A
  1. D) The gate control theory was proposed by Melzack and Wall in the mid-1960s. The concept of the gate control theory is that nonpainful input can override painful input by “closing the gate of control,” which results in suppression of pain. In medial branch block, the peripheral pain signal input is simply blocked with injected anesthetic medication.
230
Q

PA 17. C fibers are:
A) Small myelinated fibers responding to high-intensity mechanical stimulation
B) Large myelinated fibers that transmit temperature sensation
C) Small unmyelinated fibers that transmit burning pain
D) Large unmyelinated fibers that transmit noxious information from a variety of modalities

A
  1. C) A sharp, pricking, stinging pain sensation caused by a needle, pin prick, or a skin cut is transmitted by the A delta fibers. Burning pain caused by inflammation or burned skin is transmitted by C fibers.
231
Q
PA 18.  Nociceptors are:
       A)  Pacinian corpuscles
       B)  Meissner corpuscles
       C)  Merkel’s disks
       D)  Free nerve endings
A
  1. D) Nociceptors are free nerve endings that transmit the sensation of pain. There are thermal, chemical, and mechanical nociceptors for various stimuli.
232
Q

PA 19. What is the pain wind-up phenomenon?
A) Increased pain intensity by repeated stimulation
B) Recruitment of silent nociceptors after tissue injury causing increased pain intensity
C) Increased muscle tone caused by severe pain
D) Central sensitization caused by repeated stimulation of nociceptive C fibers

A
  1. D) Pain wind-up is a phenomenon caused by repeated stimulation of peripheral nerve fibers, leading to progressively increasing electrical response in the second-order neurons. The process is also termed as central sensitization, which leads to hyperalgesia, allodynia, and spontaneous pain.
233
Q
PA 20.  \_\_\_\_\_ is an oral analogue of lidocaine used in the treatment of neuropathic pain.
       A)  Mexiletine
       B)  Ketamine
       C)  Pregabalin
       D)  Amitriptyline
A

A) Mexiletine is an orally active local anesthetic, antiarrhythmic agent, that is structurally similar to lidocaine and considered an oral analogue of lidocaine for neuropathic pain treatment. Ketamine is the most commonly used NMDA antagonist for neuropathic pain. Pregabalin (Lyrica), similar to gabapentin (Neurontin), binds to the alpha2delta subunit of the voltage-dependent calcium channel in the central nervous system. It decreases the release of neurotransmitters such as glutamate, noradrenaline, and gamma-aminobutyric acid. Amitriptyline (Elavil) is a tricyclic antidepressant (TCA) used for neuropathic pain.

234
Q
PA 25.  Opioid receptors belong to the \_\_\_\_\_\_\_\_\_\_\_\_\_\_ family.
       A)  G-protein–coupled receptor
       B)  NMDA receptor
       C)  AMPA receptor
       D)  GABA receptor
A
  1. A) There are three distinct opioid receptors—mu, delta, and kappa. All of these receptors belong to the G-protein–coupled receptor family, with cAMP acting as an intracellular second messenger.
235
Q
PA 29.  Pregabalin (Lyrica) and gabapentin (Neurontin) work by blocking specific \_\_\_\_\_\_\_ channels on neurons and are preferred first-line medications for diabetic neuropathy.
       A)  Sodium
       B)  Calcium
       C)  Potassium
       D)  Magnesium
A
  1. B) Medications such as pregabalin and gabapentin work by blocking calcium channels and have been shown to decrease neuropathic pain.
236
Q
30.  The anticonvulsants carbamazepine (Tegretol) and oxcarbazepine (Trileptal) are especially effective in trigeminal neuralgia. The actions of these two drugs are mediated principally through \_\_\_\_\_ channels.
       A)  Sodium
       B)  Calcium
       C)  Potassium
       D)  Magnesium
A
  1. A) These two medications affect the sodium channels and help to reduce neuropathic pain, especially trigeminal neuralgi